SlideShare a Scribd company logo
1 of 58
Download to read offline
Class XII Chapter 11 – Three Dimensional Geometry Maths
Page 1 of 58
Website: www.vidhyarjan.com Email: contact@vidhyarjan.com Mobile: 9999 249717
Head Office: 1/3-H-A-2, Street # 6, East Azad Nagar, Delhi-110051
(One Km from ‘Welcome’ Metro Station)
Exercise 11.1
Question 1:
If a line makes angles 90°, 135°, 45° with x, y and z-axes respectively, find its direction
cosines.
Answer
Let direction cosines of the line be l, m, and n.
Therefore, the direction cosines of the line are
Question 2:
Find the direction cosines of a line which makes equal angles with the coordinate axes.
Answer
Let the direction cosines of the line make an angle α with each of the coordinate axes.
∴ l = cos α, m = cos α, n = cos α
Class XII Chapter 11 – Three Dimensional Geometry Maths
Page 2 of 58
Website: www.vidhyarjan.com Email: contact@vidhyarjan.com Mobile: 9999 249717
Head Office: 1/3-H-A-2, Street # 6, East Azad Nagar, Delhi-110051
(One Km from ‘Welcome’ Metro Station)
Thus, the direction cosines of the line, which is equally inclined to the coordinate axes,
are
Question 3:
If a line has the direction ratios −18, 12, −4, then what are its direction cosines?
Answer
If a line has direction ratios of −18, 12, and −4, then its direction cosines are
Thus, the direction cosines are .
Question 4:
Show that the points (2, 3, 4), (−1, −2, 1), (5, 8, 7) are collinear.
Answer
The given points are A (2, 3, 4), B (− 1, − 2, 1), and C (5, 8, 7).
It is known that the direction ratios of line joining the points, (x1, y1, z1) and (x2, y2, z2),
are given by, x2 − x1, y2 − y1, and z2 − z1.
The direction ratios of AB are (−1 − 2), (−2 − 3), and (1 − 4) i.e., −3, −5, and −3.
The direction ratios of BC are (5 − (− 1)), (8 − (− 2)), and (7 − 1) i.e., 6, 10, and 6.
It can be seen that the direction ratios of BC are −2 times that of AB i.e., they are
proportional.
Therefore, AB is parallel to BC. Since point B is common to both AB and BC, points A, B,
and C are collinear.
Class XII Chapter 11 – Three Dimensional Geometry Maths
Page 3 of 58
Website: www.vidhyarjan.com Email: contact@vidhyarjan.com Mobile: 9999 249717
Head Office: 1/3-H-A-2, Street # 6, East Azad Nagar, Delhi-110051
(One Km from ‘Welcome’ Metro Station)
Question 5:
Find the direction cosines of the sides of the triangle whose vertices are (3, 5, − 4), (−
1, 1, 2) and (− 5, − 5, − 2)
Answer
The vertices of ∆ABC are A (3, 5, −4), B (−1, 1, 2), and C (−5, −5, −2).
The direction ratios of side AB are (−1 − 3), (1 − 5), and (2 − (−4)) i.e., −4, −4, and 6.
Therefore, the direction cosines of AB are
The direction ratios of BC are (−5 − (−1)), (−5 − 1), and (−2 − 2) i.e., −4, −6, and −4.
Therefore, the direction cosines of BC are
The direction ratios of CA are (−5 − 3), (−5 − 5), and (−2 − (−4)) i.e., −8, −10, and 2.
Therefore, the direction cosines of AC are
Class XII Chapter 11 – Three Dimensional Geometry Maths
Page 4 of 58
Website: www.vidhyarjan.com Email: contact@vidhyarjan.com Mobile: 9999 249717
Head Office: 1/3-H-A-2, Street # 6, East Azad Nagar, Delhi-110051
(One Km from ‘Welcome’ Metro Station)
Class XII Chapter 11 – Three Dimensional Geometry Maths
Page 5 of 58
Website: www.vidhyarjan.com Email: contact@vidhyarjan.com Mobile: 9999 249717
Head Office: 1/3-H-A-2, Street # 6, East Azad Nagar, Delhi-110051
(One Km from ‘Welcome’ Metro Station)
Exercise 11.2
Question 1:
Show that the three lines with direction cosines
are mutually perpendicular.
Answer
Two lines with direction cosines, l1, m1, n1 and l2, m2, n2, are perpendicular to each
other, if l1l2 + m1m2 + n1n2 = 0
(i) For the lines with direction cosines, and , we obtain
Therefore, the lines are perpendicular.
(ii) For the lines with direction cosines, and , we obtain
Therefore, the lines are perpendicular.
(iii) For the lines with direction cosines, and , we obtain
Class XII Chapter 11 – Three Dimensional Geometry Maths
Page 6 of 58
Website: www.vidhyarjan.com Email: contact@vidhyarjan.com Mobile: 9999 249717
Head Office: 1/3-H-A-2, Street # 6, East Azad Nagar, Delhi-110051
(One Km from ‘Welcome’ Metro Station)
Therefore, the lines are perpendicular.
Thus, all the lines are mutually perpendicular.
Question 2:
Show that the line through the points (1, −1, 2) (3, 4, −2) is perpendicular to the line
through the points (0, 3, 2) and (3, 5, 6).
Answer
Let AB be the line joining the points, (1, −1, 2) and (3, 4, − 2), and CD be the line
joining the points, (0, 3, 2) and (3, 5, 6).
The direction ratios, a1, b1, c1, of AB are (3 − 1), (4 − (−1)), and (−2 − 2) i.e., 2, 5, and
−4.
The direction ratios, a2, b2, c2, of CD are (3 − 0), (5 − 3), and (6 −2) i.e., 3, 2, and 4.
AB and CD will be perpendicular to each other, if a1a2 + b1b2+ c1c2 = 0
a1a2 + b1b2+ c1c2 = 2 × 3 + 5 × 2 + (− 4) × 4
= 6 + 10 − 16
= 0
Therefore, AB and CD are perpendicular to each other.
Question 3:
Show that the line through the points (4, 7, 8) (2, 3, 4) is parallel to the line through the
points (−1, −2, 1), (1, 2, 5).
Answer
Let AB be the line through the points, (4, 7, 8) and (2, 3, 4), and CD be the line through
the points, (−1, −2, 1) and (1, 2, 5).
The directions ratios, a1, b1, c1, of AB are (2 − 4), (3 − 7), and (4 − 8) i.e., −2, −4, and
−4.
The direction ratios, a2, b2, c2, of CD are (1 − (−1)), (2 − (−2)), and (5 − 1) i.e., 2, 4,
and 4.
Class XII Chapter 11 – Three Dimensional Geometry Maths
Page 7 of 58
Website: www.vidhyarjan.com Email: contact@vidhyarjan.com Mobile: 9999 249717
Head Office: 1/3-H-A-2, Street # 6, East Azad Nagar, Delhi-110051
(One Km from ‘Welcome’ Metro Station)
AB will be parallel to CD, if
Thus, AB is parallel to CD.
Question 4:
Find the equation of the line which passes through the point (1, 2, 3) and is parallel to
the vector .
Answer
It is given that the line passes through the point A (1, 2, 3). Therefore, the position
vector through A is
It is known that the line which passes through point A and parallel to is given by
is a constant.
This is the required equation of the line.
Question 5:
Find the equation of the line in vector and in Cartesian form that passes through the
point with position vector and is in the direction .
Answer
It is given that the line passes through the point with position vector
Class XII Chapter 11 – Three Dimensional Geometry Maths
Page 8 of 58
Website: www.vidhyarjan.com Email: contact@vidhyarjan.com Mobile: 9999 249717
Head Office: 1/3-H-A-2, Street # 6, East Azad Nagar, Delhi-110051
(One Km from ‘Welcome’ Metro Station)
It is known that a line through a point with position vector and parallel to is given by
the equation,
This is the required equation of the line in vector form.
Eliminating λ, we obtain the Cartesian form equation as
This is the required equation of the given line in Cartesian form.
Question 6:
Find the Cartesian equation of the line which passes through the point
(−2, 4, −5) and parallel to the line given by
Answer
It is given that the line passes through the point (−2, 4, −5) and is parallel to
The direction ratios of the line, , are 3, 5, and 6.
The required line is parallel to
Therefore, its direction ratios are 3k, 5k, and 6k, where k ≠ 0
It is known that the equation of the line through the point (x1, y1, z1) and with direction
ratios, a, b, c, is given by
Class XII Chapter 11 – Three Dimensional Geometry Maths
Page 9 of 58
Website: www.vidhyarjan.com Email: contact@vidhyarjan.com Mobile: 9999 249717
Head Office: 1/3-H-A-2, Street # 6, East Azad Nagar, Delhi-110051
(One Km from ‘Welcome’ Metro Station)
Therefore the equation of the required line is
Question 7:
The Cartesian equation of a line is . Write its vector form.
Answer
The Cartesian equation of the line is
The given line passes through the point (5, −4, 6). The position vector of this point is
Also, the direction ratios of the given line are 3, 7, and 2.
This means that the line is in the direction of vector,
It is known that the line through position vector and in the direction of the vector is
given by the equation,
This is the required equation of the given line in vector form.
Question 8:
Find the vector and the Cartesian equations of the lines that pass through the origin and
(5, −2, 3).
Answer
The required line passes through the origin. Therefore, its position vector is given by,
The direction ratios of the line through origin and (5, −2, 3) are
(5 − 0) = 5, (−2 − 0) = −2, (3 − 0) = 3
Class XII Chapter 11 – Three Dimensional Geometry Maths
Page 10 of 58
Website: www.vidhyarjan.com Email: contact@vidhyarjan.com Mobile: 9999 249717
Head Office: 1/3-H-A-2, Street # 6, East Azad Nagar, Delhi-110051
(One Km from ‘Welcome’ Metro Station)
The line is parallel to the vector given by the equation,
The equation of the line in vector form through a point with position vector and parallel
to is,
The equation of the line through the point (x1, y1, z1) and direction ratios a, b, c is given
by,
Therefore, the equation of the required line in the Cartesian form is
Question 9:
Find the vector and the Cartesian equations of the line that passes through the points (3,
−2, −5), (3, −2, 6).
Answer
Let the line passing through the points, P (3, −2, −5) and Q (3, −2, 6), be PQ.
Since PQ passes through P (3, −2, −5), its position vector is given by,
The direction ratios of PQ are given by,
(3 − 3) = 0, (−2 + 2) = 0, (6 + 5) = 11
The equation of the vector in the direction of PQ is
The equation of PQ in vector form is given by,
The equation of PQ in Cartesian form is
Class XII Chapter 11 – Three Dimensional Geometry Maths
Page 11 of 58
Website: www.vidhyarjan.com Email: contact@vidhyarjan.com Mobile: 9999 249717
Head Office: 1/3-H-A-2, Street # 6, East Azad Nagar, Delhi-110051
(One Km from ‘Welcome’ Metro Station)
i.e.,
Question 10:
Find the angle between the following pairs of lines:
(i)
(ii) and
Answer
(i) Let Q be the angle between the given lines.
The angle between the given pairs of lines is given by,
The given lines are parallel to the vectors, and ,
respectively.
Class XII Chapter 11 – Three Dimensional Geometry Maths
Page 12 of 58
Website: www.vidhyarjan.com Email: contact@vidhyarjan.com Mobile: 9999 249717
Head Office: 1/3-H-A-2, Street # 6, East Azad Nagar, Delhi-110051
(One Km from ‘Welcome’ Metro Station)
(ii) The given lines are parallel to the vectors, and ,
respectively.
Question 11:
Find the angle between the following pairs of lines:
(i)
(ii)
i. Answer
Class XII Chapter 11 – Three Dimensional Geometry Maths
Page 13 of 58
Website: www.vidhyarjan.com Email: contact@vidhyarjan.com Mobile: 9999 249717
Head Office: 1/3-H-A-2, Street # 6, East Azad Nagar, Delhi-110051
(One Km from ‘Welcome’ Metro Station)
ii. Let and be the vectors parallel to the pair of lines,
, respectively.
and
The angle, Q, between the given pair of lines is given by the relation,
(ii) Let be the vectors parallel to the given pair of lines, and
, respectively.
Class XII Chapter 11 – Three Dimensional Geometry Maths
Page 14 of 58
Website: www.vidhyarjan.com Email: contact@vidhyarjan.com Mobile: 9999 249717
Head Office: 1/3-H-A-2, Street # 6, East Azad Nagar, Delhi-110051
(One Km from ‘Welcome’ Metro Station)
If Q is the angle between the given pair of lines, then
Question 12:
Find the values of p so the line and
are at right angles.
Answer
The given equations can be written in the standard form as
and
The direction ratios of the lines are −3, , 2 and respectively.
Two lines with direction ratios, a1, b1, c1 and a2, b2, c2, are perpendicular to each other, if
a1a2 + b1 b2 + c1c2 = 0
Class XII Chapter 11 – Three Dimensional Geometry Maths
Page 15 of 58
Website: www.vidhyarjan.com Email: contact@vidhyarjan.com Mobile: 9999 249717
Head Office: 1/3-H-A-2, Street # 6, East Azad Nagar, Delhi-110051
(One Km from ‘Welcome’ Metro Station)
Thus, the value of p is .
Question 13:
Show that the lines and are perpendicular to each other.
Answer
The equations of the given lines are and
The direction ratios of the given lines are 7, −5, 1 and 1, 2, 3 respectively.
Two lines with direction ratios, a1, b1, c1 and a2, b2, c2, are perpendicular to each other, if
a1a2 + b1 b2 + c1c2 = 0
∴ 7 × 1 + (−5) × 2 + 1 × 3
= 7 − 10 + 3
= 0
Therefore, the given lines are perpendicular to each other.
Question 14:
Find the shortest distance between the lines
Class XII Chapter 11 – Three Dimensional Geometry Maths
Page 16 of 58
Website: www.vidhyarjan.com Email: contact@vidhyarjan.com Mobile: 9999 249717
Head Office: 1/3-H-A-2, Street # 6, East Azad Nagar, Delhi-110051
(One Km from ‘Welcome’ Metro Station)
Answer
The equations of the given lines are
It is known that the shortest distance between the lines, and , is
given by,
Comparing the given equations, we obtain
Substituting all the values in equation (1), we obtain
Class XII Chapter 11 – Three Dimensional Geometry Maths
Page 17 of 58
Website: www.vidhyarjan.com Email: contact@vidhyarjan.com Mobile: 9999 249717
Head Office: 1/3-H-A-2, Street # 6, East Azad Nagar, Delhi-110051
(One Km from ‘Welcome’ Metro Station)
Therefore, the shortest distance between the two lines is units.
Question 15:
Find the shortest distance between the lines and
Answer
The given lines are and
It is known that the shortest distance between the two lines,
, is given by,
Comparing the given equations, we obtain
Class XII Chapter 11 – Three Dimensional Geometry Maths
Page 18 of 58
Website: www.vidhyarjan.com Email: contact@vidhyarjan.com Mobile: 9999 249717
Head Office: 1/3-H-A-2, Street # 6, East Azad Nagar, Delhi-110051
(One Km from ‘Welcome’ Metro Station)
Substituting all the values in equation (1), we obtain
Since distance is always non-negative, the distance between the given lines is
units.
Question 16:
Find the shortest distance between the lines whose vector equations are
Answer
The given lines are and
It is known that the shortest distance between the lines, and , is
given by,
Class XII Chapter 11 – Three Dimensional Geometry Maths
Page 19 of 58
Website: www.vidhyarjan.com Email: contact@vidhyarjan.com Mobile: 9999 249717
Head Office: 1/3-H-A-2, Street # 6, East Azad Nagar, Delhi-110051
(One Km from ‘Welcome’ Metro Station)
Comparing the given equations with and , we obtain
Substituting all the values in equation (1), we obtain
Therefore, the shortest distance between the two given lines is units.
Question 17:
Find the shortest distance between the lines whose vector equations are
Answer
The given lines are
Class XII Chapter 11 – Three Dimensional Geometry Maths
Page 20 of 58
Website: www.vidhyarjan.com Email: contact@vidhyarjan.com Mobile: 9999 249717
Head Office: 1/3-H-A-2, Street # 6, East Azad Nagar, Delhi-110051
(One Km from ‘Welcome’ Metro Station)
It is known that the shortest distance between the lines, and , is
given by,
For the given equations,
Substituting all the values in equation (3), we obtain
Therefore, the shortest distance between the lines is units.
Class XII Chapter 11 – Three Dimensional Geometry Maths
Page 21 of 58
Website: www.vidhyarjan.com Email: contact@vidhyarjan.com Mobile: 9999 249717
Head Office: 1/3-H-A-2, Street # 6, East Azad Nagar, Delhi-110051
(One Km from ‘Welcome’ Metro Station)
Exercise 11.3
Question 1:
In each of the following cases, determine the direction cosines of the normal to the plane
and the distance from the origin.
(a)z = 2 (b)
(c) (d)5y + 8 = 0
Answer
(a) The equation of the plane is z = 2 or 0x + 0y + z = 2 … (1)
The direction ratios of normal are 0, 0, and 1.
∴
Dividing both sides of equation (1) by 1, we obtain
This is of the form lx + my + nz = d, where l, m, n are the direction cosines of normal to
the plane and d is the distance of the perpendicular drawn from the origin.
Therefore, the direction cosines are 0, 0, and 1 and the distance of the plane from the
origin is 2 units.
(b) x + y + z = 1 … (1)
The direction ratios of normal are 1, 1, and 1.
∴
Dividing both sides of equation (1) by , we obtain
Class XII Chapter 11 – Three Dimensional Geometry Maths
Page 22 of 58
Website: www.vidhyarjan.com Email: contact@vidhyarjan.com Mobile: 9999 249717
Head Office: 1/3-H-A-2, Street # 6, East Azad Nagar, Delhi-110051
(One Km from ‘Welcome’ Metro Station)
This equation is of the form lx + my + nz = d, where l, m, n are the direction cosines of
normal to the plane and d is the distance of normal from the origin.
Therefore, the direction cosines of the normal are and the distance of
normal from the origin is units.
(c) 2x + 3y − z = 5 … (1)
The direction ratios of normal are 2, 3, and −1.
Dividing both sides of equation (1) by , we obtain
This equation is of the form lx + my + nz = d, where l, m, n are the direction cosines of
normal to the plane and d is the distance of normal from the origin.
Therefore, the direction cosines of the normal to the plane are and
the distance of normal from the origin is units.
(d) 5y + 8 = 0
⇒ 0x − 5y + 0z = 8 … (1)
The direction ratios of normal are 0, −5, and 0.
Dividing both sides of equation (1) by 5, we obtain
Class XII Chapter 11 – Three Dimensional Geometry Maths
Page 23 of 58
Website: www.vidhyarjan.com Email: contact@vidhyarjan.com Mobile: 9999 249717
Head Office: 1/3-H-A-2, Street # 6, East Azad Nagar, Delhi-110051
(One Km from ‘Welcome’ Metro Station)
This equation is of the form lx + my + nz = d, where l, m, n are the direction cosines of
normal to the plane and d is the distance of normal from the origin.
Therefore, the direction cosines of the normal to the plane are 0, −1, and 0 and the
distance of normal from the origin is units.
Question 2:
Find the vector equation of a plane which is at a distance of 7 units from the origin and
normal to the vector .
Answer
The normal vector is,
It is known that the equation of the plane with position vector is given by,
This is the vector equation of the required plane.
Question 3:
Find the Cartesian equation of the following planes:
(a) (b)
(c)
Answer
(a) It is given that equation of the plane is
For any arbitrary point P (x, y, z) on the plane, position vector is given by,
Class XII Chapter 11 – Three Dimensional Geometry Maths
Page 24 of 58
Website: www.vidhyarjan.com Email: contact@vidhyarjan.com Mobile: 9999 249717
Head Office: 1/3-H-A-2, Street # 6, East Azad Nagar, Delhi-110051
(One Km from ‘Welcome’ Metro Station)
Substituting the value of in equation (1), we obtain
This is the Cartesian equation of the plane.
(b)
For any arbitrary point P (x, y, z) on the plane, position vector is given by,
Substituting the value of in equation (1), we obtain
This is the Cartesian equation of the plane.
(c)
For any arbitrary point P (x, y, z) on the plane, position vector is given by,
Substituting the value of in equation (1), we obtain
This is the Cartesian equation of the given plane.
Question 4:
In the following cases, find the coordinates of the foot of the perpendicular drawn from
the origin.
(a) (b)
(c) (d)
Answer
(a) Let the coordinates of the foot of perpendicular P from the origin to the plane be
(x1, y1, z1).
Class XII Chapter 11 – Three Dimensional Geometry Maths
Page 25 of 58
Website: www.vidhyarjan.com Email: contact@vidhyarjan.com Mobile: 9999 249717
Head Office: 1/3-H-A-2, Street # 6, East Azad Nagar, Delhi-110051
(One Km from ‘Welcome’ Metro Station)
2x + 3y + 4z − 12 = 0
⇒ 2x + 3y + 4z = 12 … (1)
The direction ratios of normal are 2, 3, and 4.
Dividing both sides of equation (1) by , we obtain
This equation is of the form lx + my + nz = d, where l, m, n are the direction cosines of
normal to the plane and d is the distance of normal from the origin.
The coordinates of the foot of the perpendicular are given by
(ld, md, nd).
Therefore, the coordinates of the foot of the perpendicular are
(b) Let the coordinates of the foot of perpendicular P from the origin to the plane be (x1,
y1, z1).
⇒ … (1)
The direction ratios of the normal are 0, 3, and 4.
Dividing both sides of equation (1) by 5, we obtain
Class XII Chapter 11 – Three Dimensional Geometry Maths
Page 26 of 58
Website: www.vidhyarjan.com Email: contact@vidhyarjan.com Mobile: 9999 249717
Head Office: 1/3-H-A-2, Street # 6, East Azad Nagar, Delhi-110051
(One Km from ‘Welcome’ Metro Station)
This equation is of the form lx + my + nz = d, where l, m, n are the direction cosines of
normal to the plane and d is the distance of normal from the origin.
The coordinates of the foot of the perpendicular are given by
(ld, md, nd).
Therefore, the coordinates of the foot of the perpendicular are
(c) Let the coordinates of the foot of perpendicular P from the origin to the plane be (x1,
y1, z1).
… (1)
The direction ratios of the normal are 1, 1, and 1.
Dividing both sides of equation (1) by , we obtain
This equation is of the form lx + my + nz = d, where l, m, n are the direction cosines of
normal to the plane and d is the distance of normal from the origin.
The coordinates of the foot of the perpendicular are given by
(ld, md, nd).
Therefore, the coordinates of the foot of the perpendicular are
(d) Let the coordinates of the foot of perpendicular P from the origin to the plane be (x1,
y1, z1).
⇒ 0x − 5y + 0z = 8 … (1)
The direction ratios of the normal are 0, −5, and 0.
Class XII Chapter 11 – Three Dimensional Geometry Maths
Page 27 of 58
Website: www.vidhyarjan.com Email: contact@vidhyarjan.com Mobile: 9999 249717
Head Office: 1/3-H-A-2, Street # 6, East Azad Nagar, Delhi-110051
(One Km from ‘Welcome’ Metro Station)
Dividing both sides of equation (1) by 5, we obtain
This equation is of the form lx + my + nz = d, where l, m, n are the direction cosines of
normal to the plane and d is the distance of normal from the origin.
The coordinates of the foot of the perpendicular are given by
(ld, md, nd).
Therefore, the coordinates of the foot of the perpendicular are
Question 5:
Find the vector and Cartesian equation of the planes
(a) that passes through the point (1, 0, −2) and the normal to the plane is .
(b) that passes through the point (1, 4, 6) and the normal vector to the plane is
.
Answer
(a) The position vector of point (1, 0, −2) is
The normal vector perpendicular to the plane is
The vector equation of the plane is given by,
is the position vector of any point P (x, y, z) in the plane.
Therefore, equation (1) becomes
Class XII Chapter 11 – Three Dimensional Geometry Maths
Page 28 of 58
Website: www.vidhyarjan.com Email: contact@vidhyarjan.com Mobile: 9999 249717
Head Office: 1/3-H-A-2, Street # 6, East Azad Nagar, Delhi-110051
(One Km from ‘Welcome’ Metro Station)
This is the Cartesian equation of the required plane.
(b) The position vector of the point (1, 4, 6) is
The normal vector perpendicular to the plane is
The vector equation of the plane is given by,
is the position vector of any point P (x, y, z) in the plane.
Therefore, equation (1) becomes
This is the Cartesian equation of the required plane.
Question 6:
Find the equations of the planes that passes through three points.
(a) (1, 1, −1), (6, 4, −5), (−4, −2, 3)
(b) (1, 1, 0), (1, 2, 1), (−2, 2, −1)
Answer
(a) The given points are A (1, 1, −1), B (6, 4, −5), and C (−4, −2, 3).
Class XII Chapter 11 – Three Dimensional Geometry Maths
Page 29 of 58
Website: www.vidhyarjan.com Email: contact@vidhyarjan.com Mobile: 9999 249717
Head Office: 1/3-H-A-2, Street # 6, East Azad Nagar, Delhi-110051
(One Km from ‘Welcome’ Metro Station)
Since A, B, C are collinear points, there will be infinite number of planes passing through
the given points.
(b) The given points are A (1, 1, 0), B (1, 2, 1), and C (−2, 2, −1).
Therefore, a plane will pass through the points A, B, and C.
It is known that the equation of the plane through the points, , and
, is
This is the Cartesian equation of the required plane.
Question 7:
Find the intercepts cut off by the plane
Answer
Class XII Chapter 11 – Three Dimensional Geometry Maths
Page 30 of 58
Website: www.vidhyarjan.com Email: contact@vidhyarjan.com Mobile: 9999 249717
Head Office: 1/3-H-A-2, Street # 6, East Azad Nagar, Delhi-110051
(One Km from ‘Welcome’ Metro Station)
Dividing both sides of equation (1) by 5, we obtain
It is known that the equation of a plane in intercept form is , where a, b, c
are the intercepts cut off by the plane at x, y, and z axes respectively.
Therefore, for the given equation,
Thus, the intercepts cut off by the plane are .
Question 8:
Find the equation of the plane with intercept 3 on the y-axis and parallel to ZOX plane.
Answer
The equation of the plane ZOX is
y = 0
Any plane parallel to it is of the form, y = a
Since the y-intercept of the plane is 3,
∴ a = 3
Thus, the equation of the required plane is y = 3
Question 9:
Find the equation of the plane through the intersection of the planes
and and the point (2, 2, 1)
Answer
Class XII Chapter 11 – Three Dimensional Geometry Maths
Page 31 of 58
Website: www.vidhyarjan.com Email: contact@vidhyarjan.com Mobile: 9999 249717
Head Office: 1/3-H-A-2, Street # 6, East Azad Nagar, Delhi-110051
(One Km from ‘Welcome’ Metro Station)
The equation of any plane through the intersection of the planes,
3x − y + 2z − 4 = 0 and x + y + z − 2 = 0, is
The plane passes through the point (2, 2, 1). Therefore, this point will satisfy equation
(1).
Substituting in equation (1), we obtain
This is the required equation of the plane.
Question 10:
Find the vector equation of the plane passing through the intersection of the planes
and through the point (2, 1, 3)
Answer
The equations of the planes are
The equation of any plane through the intersection of the planes given in equations (1)
and (2) is given by,
, where
Class XII Chapter 11 – Three Dimensional Geometry Maths
Page 32 of 58
Website: www.vidhyarjan.com Email: contact@vidhyarjan.com Mobile: 9999 249717
Head Office: 1/3-H-A-2, Street # 6, East Azad Nagar, Delhi-110051
(One Km from ‘Welcome’ Metro Station)
The plane passes through the point (2, 1, 3). Therefore, its position vector is given by,
Substituting in equation (3), we obtain
Substituting in equation (3), we obtain
This is the vector equation of the required plane.
Question 11:
Find the equation of the plane through the line of intersection of the planes
and which is perpendicular to the plane
Answer
The equation of the plane through the intersection of the planes, and
, is
The direction ratios, a1, b1, c1, of this plane are (2λ + 1), (3λ + 1), and (4λ + 1).
The plane in equation (1) is perpendicular to
Class XII Chapter 11 – Three Dimensional Geometry Maths
Page 33 of 58
Website: www.vidhyarjan.com Email: contact@vidhyarjan.com Mobile: 9999 249717
Head Office: 1/3-H-A-2, Street # 6, East Azad Nagar, Delhi-110051
(One Km from ‘Welcome’ Metro Station)
Its direction ratios, a2, b2, c2, are 1, −1, and 1.
Since the planes are perpendicular,
Substituting in equation (1), we obtain
This is the required equation of the plane.
Question 12:
Find the angle between the planes whose vector equations are
and .
Answer
The equations of the given planes are and
It is known that if and are normal to the planes, and , then the
angle between them, Q, is given by,
Here,
Class XII Chapter 11 – Three Dimensional Geometry Maths
Page 34 of 58
Website: www.vidhyarjan.com Email: contact@vidhyarjan.com Mobile: 9999 249717
Head Office: 1/3-H-A-2, Street # 6, East Azad Nagar, Delhi-110051
(One Km from ‘Welcome’ Metro Station)
Substituting the value of , in equation (1), we obtain
Question 13:
In the following cases, determine whether the given planes are parallel or perpendicular,
and in case they are neither, find the angles between them.
(a)
(b)
(c)
(d)
(e)
Answer
The direction ratios of normal to the plane, , are a1, b1, c1 and
.
The angle between L1 and L2 is given by,
(a) The equations of the planes are 7x + 5y + 6z + 30 = 0 and
3x − y − 10z + 4 = 0
Here, a1 = 7, b1 =5, c1 = 6
Class XII Chapter 11 – Three Dimensional Geometry Maths
Page 35 of 58
Website: www.vidhyarjan.com Email: contact@vidhyarjan.com Mobile: 9999 249717
Head Office: 1/3-H-A-2, Street # 6, East Azad Nagar, Delhi-110051
(One Km from ‘Welcome’ Metro Station)
Therefore, the given planes are not perpendicular.
It can be seen that,
Therefore, the given planes are not parallel.
The angle between them is given by,
(b) The equations of the planes are and
Here, and
Thus, the given planes are perpendicular to each other.
(c) The equations of the given planes are and
Here, and
Thus, the given planes are not perpendicular to each other.
Class XII Chapter 11 – Three Dimensional Geometry Maths
Page 36 of 58
Website: www.vidhyarjan.com Email: contact@vidhyarjan.com Mobile: 9999 249717
Head Office: 1/3-H-A-2, Street # 6, East Azad Nagar, Delhi-110051
(One Km from ‘Welcome’ Metro Station)
∴
Thus, the given planes are parallel to each other.
(d) The equations of the planes are and
Here, and
∴
Thus, the given lines are parallel to each other.
(e) The equations of the given planes are and
Here, and
Therefore, the given lines are not perpendicular to each other.
∴
Therefore, the given lines are not parallel to each other.
The angle between the planes is given by,
Class XII Chapter 11 – Three Dimensional Geometry Maths
Page 37 of 58
Website: www.vidhyarjan.com Email: contact@vidhyarjan.com Mobile: 9999 249717
Head Office: 1/3-H-A-2, Street # 6, East Azad Nagar, Delhi-110051
(One Km from ‘Welcome’ Metro Station)
Question 14:
In the following cases, find the distance of each of the given points from the
corresponding given plane.
Point Plane
(a) (0, 0, 0)
(b) (3, −2, 1)
(c) (2, 3, −5)
(d) (−6, 0, 0)
Answer
It is known that the distance between a point, p(x1, y1, z1), and a plane, Ax + By + Cz =
D, is given by,
(a) The given point is (0, 0, 0) and the plane is
(b) The given point is (3, − 2, 1) and the plane is
∴
(c) The given point is (2, 3, −5) and the plane is
(d) The given point is (−6, 0, 0) and the plane is
Class XII Chapter 11 – Three Dimensional Geometry Maths
Page 38 of 58
Website: www.vidhyarjan.com Email: contact@vidhyarjan.com Mobile: 9999 249717
Head Office: 1/3-H-A-2, Street # 6, East Azad Nagar, Delhi-110051
(One Km from ‘Welcome’ Metro Station)
Class XII Chapter 11 – Three Dimensional Geometry Maths
Page 39 of 58
Website: www.vidhyarjan.com Email: contact@vidhyarjan.com Mobile: 9999 249717
Head Office: 1/3-H-A-2, Street # 6, East Azad Nagar, Delhi-110051
(One Km from ‘Welcome’ Metro Station)
Miscellaneous Solutions
Question 1:
Show that the line joining the origin to the point (2, 1, 1) is perpendicular to the line
determined by the points (3, 5, −1), (4, 3, −1).
Answer
Let OA be the line joining the origin, O (0, 0, 0), and the point, A (2, 1, 1).
Also, let BC be the line joining the points, B (3, 5, −1) and C (4, 3, −1).
The direction ratios of OA are 2, 1, and 1 and of BC are (4 − 3) = 1, (3 − 5) = −2, and
(−1 + 1) = 0
OA is perpendicular to BC, if a1a2 + b1b2 + c1c2 = 0
∴ a1a2 + b1b2 + c1c2 = 2 × 1 + 1 (−2) + 1 ×0 = 2 − 2 = 0
Thus, OA is perpendicular to BC.
Question 2:
If l1, m1, n1 and l2, m2, n2 are the direction cosines of two mutually perpendicular lines,
show that the direction cosines of the line perpendicular to both of these are m1n2 −
m2n1, n1l2 − n2l1, l1m2 − l2m1.
Answer
It is given that l1, m1, n1 and l2, m2, n2 are the direction cosines of two mutually
perpendicular lines. Therefore,
Let l, m, n be the direction cosines of the line which is perpendicular to the line with
direction cosines l1, m1, n1 and l2, m2, n2.
Class XII Chapter 11 – Three Dimensional Geometry Maths
Page 40 of 58
Website: www.vidhyarjan.com Email: contact@vidhyarjan.com Mobile: 9999 249717
Head Office: 1/3-H-A-2, Street # 6, East Azad Nagar, Delhi-110051
(One Km from ‘Welcome’ Metro Station)
l, m, n are the direction cosines of the line.
∴l2
+ m2
+ n2
= 1 … (5)
It is known that,
∴
Substituting the values from equations (5) and (6) in equation (4), we obtain
Thus, the direction cosines of the required line are
Class XII Chapter 11 – Three Dimensional Geometry Maths
Page 41 of 58
Website: www.vidhyarjan.com Email: contact@vidhyarjan.com Mobile: 9999 249717
Head Office: 1/3-H-A-2, Street # 6, East Azad Nagar, Delhi-110051
(One Km from ‘Welcome’ Metro Station)
Question 3:
Find the angle between the lines whose direction ratios are a, b, c and b − c,
c − a, a − b.
Answer
The angle Q between the lines with direction cosines, a, b, c and b − c, c − a,
a − b, is given by,
Thus, the angle between the lines is 90°.
Question 4:
Find the equation of a line parallel to x-axis and passing through the origin.
Answer
The line parallel to x-axis and passing through the origin is x-axis itself.
Let A be a point on x-axis. Therefore, the coordinates of A are given by (a, 0, 0), where
a ∈ R.
Direction ratios of OA are (a − 0) = a, 0, 0
The equation of OA is given by,
Thus, the equation of line parallel to x-axis and passing through origin is
Class XII Chapter 11 – Three Dimensional Geometry Maths
Page 42 of 58
Website: www.vidhyarjan.com Email: contact@vidhyarjan.com Mobile: 9999 249717
Head Office: 1/3-H-A-2, Street # 6, East Azad Nagar, Delhi-110051
(One Km from ‘Welcome’ Metro Station)
Question 5:
If the coordinates of the points A, B, C, D be (1, 2, 3), (4, 5, 7), (−4, 3, −6) and (2, 9,
2) respectively, then find the angle between the lines AB and CD.
Answer
The coordinates of A, B, C, and D are (1, 2, 3), (4, 5, 7), (−4, 3, −6), and
(2, 9, 2) respectively.
The direction ratios of AB are (4 − 1) = 3, (5 − 2) = 3, and (7 − 3) = 4
The direction ratios of CD are (2 −(− 4)) = 6, (9 − 3) = 6, and (2 −(−6)) = 8
It can be seen that,
Therefore, AB is parallel to CD.
Thus, the angle between AB and CD is either 0° or 180°.
sQuestion 6:
If the lines and are perpendicular, find the value
of k.
Answer
The direction of ratios of the lines, and , are −3,
2k, 2 and 3k, 1, −5 respectively.
It is known that two lines with direction ratios, a1, b1, c1 and a2, b2, c2, are
perpendicular, if a1a2 + b1b2 + c1c2 = 0
Class XII Chapter 11 – Three Dimensional Geometry Maths
Page 43 of 58
Website: www.vidhyarjan.com Email: contact@vidhyarjan.com Mobile: 9999 249717
Head Office: 1/3-H-A-2, Street # 6, East Azad Nagar, Delhi-110051
(One Km from ‘Welcome’ Metro Station)
Therefore, for , the given lines are perpendicular to each other.
Question 7:
Find the vector equation of the plane passing through (1, 2, 3) and perpendicular to the
plane
Answer
The position vector of the point (1, 2, 3) is
The direction ratios of the normal to the plane, , are 1, 2, and −5
and the normal vector is
The equation of a line passing through a point and perpendicular to the given plane is
given by,
Question 8:
Find the equation of the plane passing through (a, b, c) and parallel to the plane
Answer
Any plane parallel to the plane, , is of the form
The plane passes through the point (a, b, c). Therefore, the position vector of this
point is
Therefore, equation (1) becomes
Class XII Chapter 11 – Three Dimensional Geometry Maths
Page 44 of 58
Website: www.vidhyarjan.com Email: contact@vidhyarjan.com Mobile: 9999 249717
Head Office: 1/3-H-A-2, Street # 6, East Azad Nagar, Delhi-110051
(One Km from ‘Welcome’ Metro Station)
Substituting in equation (1), we obtain
This is the vector equation of the required plane.
Substituting in equation (2), we obtain
Question 9:
Find the shortest distance between lines
and .
Answer
The given lines are
It is known that the shortest distance between two lines, and , is
given by
Comparing to equations (1) and (2), we obtain
Class XII Chapter 11 – Three Dimensional Geometry Maths
Page 45 of 58
Website: www.vidhyarjan.com Email: contact@vidhyarjan.com Mobile: 9999 249717
Head Office: 1/3-H-A-2, Street # 6, East Azad Nagar, Delhi-110051
(One Km from ‘Welcome’ Metro Station)
Substituting all the values in equation (1), we obtain
Therefore, the shortest distance between the two given lines is 9 units.
Question 10:
Find the coordinates of the point where the line through (5, 1, 6) and
(3, 4, 1) crosses the YZ-plane
Answer
It is known that the equation of the line passing through the points, (x1, y1, z1) and (x2,
y2, z2), is
The line passing through the points, (5, 1, 6) and (3, 4, 1), is given by,
Any point on the line is of the form (5 − 2k, 3k + 1, 6 −5k).
The equation of YZ-plane is x = 0
Since the line passes through YZ-plane,
5 − 2k = 0
Class XII Chapter 11 – Three Dimensional Geometry Maths
Page 46 of 58
Website: www.vidhyarjan.com Email: contact@vidhyarjan.com Mobile: 9999 249717
Head Office: 1/3-H-A-2, Street # 6, East Azad Nagar, Delhi-110051
(One Km from ‘Welcome’ Metro Station)
Therefore, the required point is .
Question 11:
Find the coordinates of the point where the line through (5, 1, 6) and
(3, 4, 1) crosses the ZX − plane.
Answer
It is known that the equation of the line passing through the points, (x1, y1, z1) and (x2,
y2, z2), is
The line passing through the points, (5, 1, 6) and (3, 4, 1), is given by,
Any point on the line is of the form (5 − 2k, 3k + 1, 6 −5k).
Since the line passes through ZX-plane,
Therefore, the required point is .
Class XII Chapter 11 – Three Dimensional Geometry Maths
Page 47 of 58
Website: www.vidhyarjan.com Email: contact@vidhyarjan.com Mobile: 9999 249717
Head Office: 1/3-H-A-2, Street # 6, East Azad Nagar, Delhi-110051
(One Km from ‘Welcome’ Metro Station)
Question 12:
Find the coordinates of the point where the line through (3, −4, −5) and (2, − 3, 1)
crosses the plane 2x + y + z = 7).
Answer
It is known that the equation of the line through the points, (x1, y1, z1) and (x2, y2, z2), is
Since the line passes through the points, (3, −4, −5) and (2, −3, 1), its equation is
given by,
Therefore, any point on the line is of the form (3 − k, k − 4, 6k − 5).
This point lies on the plane, 2x + y + z = 7
∴ 2 (3 − k) + (k − 4) + (6k − 5) = 7
Hence, the coordinates of the required point are (3 − 2, 2 − 4, 6 × 2 − 5) i.e.,
(1, −2, 7).
Question 13:
Find the equation of the plane passing through the point (−1, 3, 2) and perpendicular to
each of the planes x + 2y + 3z = 5 and 3x + 3y + z = 0.
Answer
The equation of the plane passing through the point (−1, 3, 2) is
a (x + 1) + b (y − 3) + c (z − 2) = 0 … (1)
where, a, b, c are the direction ratios of normal to the plane.
Class XII Chapter 11 – Three Dimensional Geometry Maths
Page 48 of 58
Website: www.vidhyarjan.com Email: contact@vidhyarjan.com Mobile: 9999 249717
Head Office: 1/3-H-A-2, Street # 6, East Azad Nagar, Delhi-110051
(One Km from ‘Welcome’ Metro Station)
It is known that two planes, and , are
perpendicular, if
Plane (1) is perpendicular to the plane, x + 2y + 3z = 5
Also, plane (1) is perpendicular to the plane, 3x + 3y + z = 0
From equations (2) and (3), we obtain
Substituting the values of a, b, and c in equation (1), we obtain
This is the required equation of the plane.
Question 14:
If the points (1, 1, p) and (−3, 0, 1) be equidistant from the plane
, then find the value of p.
Answer
The position vector through the point (1, 1, p) is
Similarly, the position vector through the point (−3, 0, 1) is
Class XII Chapter 11 – Three Dimensional Geometry Maths
Page 49 of 58
Website: www.vidhyarjan.com Email: contact@vidhyarjan.com Mobile: 9999 249717
Head Office: 1/3-H-A-2, Street # 6, East Azad Nagar, Delhi-110051
(One Km from ‘Welcome’ Metro Station)
The equation of the given plane is
It is known that the perpendicular distance between a point whose position vector is
and the plane, is given by,
Here, and d
Therefore, the distance between the point (1, 1, p) and the given plane is
Similarly, the distance between the point (−3, 0, 1) and the given plane is
It is given that the distance between the required plane and the points, (1, 1, p) and
(−3, 0, 1), is equal.
∴ D1 = D2
Class XII Chapter 11 – Three Dimensional Geometry Maths
Page 50 of 58
Website: www.vidhyarjan.com Email: contact@vidhyarjan.com Mobile: 9999 249717
Head Office: 1/3-H-A-2, Street # 6, East Azad Nagar, Delhi-110051
(One Km from ‘Welcome’ Metro Station)
Question 15:
Find the equation of the plane passing through the line of intersection of the planes
and and parallel to x-axis.
Answer
The given planes are
The equation of any plane passing through the line of intersection of these planes is
Its direction ratios are (2λ + 1), (3λ + 1), and (1 − λ).
The required plane is parallel to x-axis. Therefore, its normal is perpendicular to x-axis.
The direction ratios of x-axis are 1, 0, and 0.
Substituting in equation (1), we obtain
Class XII Chapter 11 – Three Dimensional Geometry Maths
Page 51 of 58
Website: www.vidhyarjan.com Email: contact@vidhyarjan.com Mobile: 9999 249717
Head Office: 1/3-H-A-2, Street # 6, East Azad Nagar, Delhi-110051
(One Km from ‘Welcome’ Metro Station)
Therefore, its Cartesian equation is y − 3z + 6 = 0
This is the equation of the required plane.
Question 16:
If O be the origin and the coordinates of P be (1, 2, −3), then find the equation of the
plane passing through P and perpendicular to OP.
Answer
The coordinates of the points, O and P, are (0, 0, 0) and (1, 2, −3) respectively.
Therefore, the direction ratios of OP are (1 − 0) = 1, (2 − 0) = 2, and (−3 − 0) = −3
It is known that the equation of the plane passing through the point (x1, y1 z1) is
where, a, b, and c are the direction ratios of normal.
Here, the direction ratios of normal are 1, 2, and −3 and the point P is (1, 2, −3).
Thus, the equation of the required plane is
Question 17:
Find the equation of the plane which contains the line of intersection of the planes
, and which is perpendicular to the plane
.
Answer
The equations of the given planes are
The equation of the plane passing through the line intersection of the plane given in
equation (1) and equation (2) is
Class XII Chapter 11 – Three Dimensional Geometry Maths
Page 52 of 58
Website: www.vidhyarjan.com Email: contact@vidhyarjan.com Mobile: 9999 249717
Head Office: 1/3-H-A-2, Street # 6, East Azad Nagar, Delhi-110051
(One Km from ‘Welcome’ Metro Station)
The plane in equation (3) is perpendicular to the plane,
Substituting in equation (3), we obtain
This is the vector equation of the required plane.
The Cartesian equation of this plane can be obtained by substituting in
equation (3).
Question 18:
Find the distance of the point (−1, −5, −10) from the point of intersection of the line
and the plane .
Answer
The equation of the given line is
The equation of the given plane is
Substituting the value of from equation (1) in equation (2), we obtain
Class XII Chapter 11 – Three Dimensional Geometry Maths
Page 53 of 58
Website: www.vidhyarjan.com Email: contact@vidhyarjan.com Mobile: 9999 249717
Head Office: 1/3-H-A-2, Street # 6, East Azad Nagar, Delhi-110051
(One Km from ‘Welcome’ Metro Station)
Substituting this value in equation (1), we obtain the equation of the line as
This means that the position vector of the point of intersection of the line and the plane
is
This shows that the point of intersection of the given line and plane is given by the
coordinates, (2, −1, 2). The point is (−1, −5, −10).
The distance d between the points, (2, −1, 2) and (−1, −5, −10), is
Question 19:
Find the vector equation of the line passing through (1, 2, 3) and parallel to the planes
and .
Answer
Let the required line be parallel to vector given by,
The position vector of the point (1, 2, 3) is
The equation of line passing through (1, 2, 3) and parallel to is given by,
The equations of the given planes are
Class XII Chapter 11 – Three Dimensional Geometry Maths
Page 54 of 58
Website: www.vidhyarjan.com Email: contact@vidhyarjan.com Mobile: 9999 249717
Head Office: 1/3-H-A-2, Street # 6, East Azad Nagar, Delhi-110051
(One Km from ‘Welcome’ Metro Station)
The line in equation (1) and plane in equation (2) are parallel. Therefore, the normal to
the plane of equation (2) and the given line are perpendicular.
From equations (4) and (5), we obtain
Therefore, the direction ratios of are −3, 5, and 4.
Substituting the value of in equation (1), we obtain
This is the equation of the required line.
Question 20:
Find the vector equation of the line passing through the point (1, 2, − 4) and
perpendicular to the two lines:
Answer
Let the required line be parallel to the vector given by,
Class XII Chapter 11 – Three Dimensional Geometry Maths
Page 55 of 58
Website: www.vidhyarjan.com Email: contact@vidhyarjan.com Mobile: 9999 249717
Head Office: 1/3-H-A-2, Street # 6, East Azad Nagar, Delhi-110051
(One Km from ‘Welcome’ Metro Station)
The position vector of the point (1, 2, − 4) is
The equation of the line passing through (1, 2, −4) and parallel to vector is
The equations of the lines are
Line (1) and line (2) are perpendicular to each other.
Also, line (1) and line (3) are perpendicular to each other.
From equations (4) and (5), we obtain
∴Direction ratios of are 2, 3, and 6.
Substituting in equation (1), we obtain
This is the equation of the required line.
Class XII Chapter 11 – Three Dimensional Geometry Maths
Page 56 of 58
Website: www.vidhyarjan.com Email: contact@vidhyarjan.com Mobile: 9999 249717
Head Office: 1/3-H-A-2, Street # 6, East Azad Nagar, Delhi-110051
(One Km from ‘Welcome’ Metro Station)
Question 21:
Prove that if a plane has the intercepts a, b, c and is at a distance of P units from the
origin, then
Answer
The equation of a plane having intercepts a, b, c with x, y, and z axes respectively is
given by,
The distance (p) of the plane from the origin is given by,
Question 22:
Distance between the two planes: and is
(A)2 units (B)4 units (C)8 units
(D)
Answer
The equations of the planes are
Class XII Chapter 11 – Three Dimensional Geometry Maths
Page 57 of 58
Website: www.vidhyarjan.com Email: contact@vidhyarjan.com Mobile: 9999 249717
Head Office: 1/3-H-A-2, Street # 6, East Azad Nagar, Delhi-110051
(One Km from ‘Welcome’ Metro Station)
It can be seen that the given planes are parallel.
It is known that the distance between two parallel planes, ax + by + cz = d1 and ax + by
+ cz = d2, is given by,
Thus, the distance between the lines is units.
Hence, the correct answer is D.
Question 23:
The planes: 2x − y + 4z = 5 and 5x − 2.5y + 10z = 6 are
(A) Perpendicular (B) Parallel (C) intersect y-axis
(C) passes through
Answer
The equations of the planes are
2x − y + 4z = 5 … (1)
5x − 2.5y + 10z = 6 … (2)
It can be seen that,
Class XII Chapter 11 – Three Dimensional Geometry Maths
Page 58 of 58
Website: www.vidhyarjan.com Email: contact@vidhyarjan.com Mobile: 9999 249717
Head Office: 1/3-H-A-2, Street # 6, East Azad Nagar, Delhi-110051
(One Km from ‘Welcome’ Metro Station)
∴
Therefore, the given planes are parallel.
Hence, the correct answer is B.

More Related Content

What's hot

Chromatic graph theory
Chromatic graph theoryChromatic graph theory
Chromatic graph theoryjotasmall
 
Compiled and solved problems in geometry and trigonometry,F.Smaradanhe
Compiled and solved problems in geometry and trigonometry,F.SmaradanheCompiled and solved problems in geometry and trigonometry,F.Smaradanhe
Compiled and solved problems in geometry and trigonometry,F.SmaradanheΘανάσης Δρούγας
 
BSC_COMPUTER _SCIENCE_UNIT-3_DISCRETE MATHEMATICS
BSC_COMPUTER _SCIENCE_UNIT-3_DISCRETE MATHEMATICSBSC_COMPUTER _SCIENCE_UNIT-3_DISCRETE MATHEMATICS
BSC_COMPUTER _SCIENCE_UNIT-3_DISCRETE MATHEMATICSRai University
 
Introductory maths analysis chapter 03 official
Introductory maths analysis   chapter 03 officialIntroductory maths analysis   chapter 03 official
Introductory maths analysis chapter 03 officialEvert Sandye Taasiringan
 
linear equation system with 2 and 3 variables
linear equation system with 2 and 3 variableslinear equation system with 2 and 3 variables
linear equation system with 2 and 3 variablesWanda Sari
 
Applications of graph theory
                      Applications of graph theory                      Applications of graph theory
Applications of graph theoryNilaNila16
 
Matrix Algebra : Mathematics for Business
Matrix Algebra : Mathematics for BusinessMatrix Algebra : Mathematics for Business
Matrix Algebra : Mathematics for BusinessKhan Tanjeel Ahmed
 
introduction_to_euclids_geometry
 introduction_to_euclids_geometry introduction_to_euclids_geometry
introduction_to_euclids_geometryKripi Mehra
 
CLASS XI RELATION & FUNCTION M WORKSHEET
CLASS XI RELATION & FUNCTION M WORKSHEETCLASS XI RELATION & FUNCTION M WORKSHEET
CLASS XI RELATION & FUNCTION M WORKSHEETMishal Chauhan
 
Diploma_Semester-II_Advanced Mathematics_Complex number
Diploma_Semester-II_Advanced Mathematics_Complex numberDiploma_Semester-II_Advanced Mathematics_Complex number
Diploma_Semester-II_Advanced Mathematics_Complex numberRai University
 

What's hot (19)

Chromatic graph theory
Chromatic graph theoryChromatic graph theory
Chromatic graph theory
 
Compiled and solved problems in geometry and trigonometry,F.Smaradanhe
Compiled and solved problems in geometry and trigonometry,F.SmaradanheCompiled and solved problems in geometry and trigonometry,F.Smaradanhe
Compiled and solved problems in geometry and trigonometry,F.Smaradanhe
 
BSC_COMPUTER _SCIENCE_UNIT-3_DISCRETE MATHEMATICS
BSC_COMPUTER _SCIENCE_UNIT-3_DISCRETE MATHEMATICSBSC_COMPUTER _SCIENCE_UNIT-3_DISCRETE MATHEMATICS
BSC_COMPUTER _SCIENCE_UNIT-3_DISCRETE MATHEMATICS
 
Data Structures Chapter-2
Data Structures Chapter-2Data Structures Chapter-2
Data Structures Chapter-2
 
Test of consistency
Test of consistencyTest of consistency
Test of consistency
 
Data structures chapter 1
Data structures chapter  1Data structures chapter  1
Data structures chapter 1
 
Introductory maths analysis chapter 03 official
Introductory maths analysis   chapter 03 officialIntroductory maths analysis   chapter 03 official
Introductory maths analysis chapter 03 official
 
Matrix
MatrixMatrix
Matrix
 
Data Structures Chapter-4
Data Structures Chapter-4Data Structures Chapter-4
Data Structures Chapter-4
 
linear equation system with 2 and 3 variables
linear equation system with 2 and 3 variableslinear equation system with 2 and 3 variables
linear equation system with 2 and 3 variables
 
Matrices
MatricesMatrices
Matrices
 
U36123129
U36123129U36123129
U36123129
 
Applications of graph theory
                      Applications of graph theory                      Applications of graph theory
Applications of graph theory
 
Matrix Algebra : Mathematics for Business
Matrix Algebra : Mathematics for BusinessMatrix Algebra : Mathematics for Business
Matrix Algebra : Mathematics for Business
 
introduction_to_euclids_geometry
 introduction_to_euclids_geometry introduction_to_euclids_geometry
introduction_to_euclids_geometry
 
Matrices
MatricesMatrices
Matrices
 
CLASS XI RELATION & FUNCTION M WORKSHEET
CLASS XI RELATION & FUNCTION M WORKSHEETCLASS XI RELATION & FUNCTION M WORKSHEET
CLASS XI RELATION & FUNCTION M WORKSHEET
 
Diploma_Semester-II_Advanced Mathematics_Complex number
Diploma_Semester-II_Advanced Mathematics_Complex numberDiploma_Semester-II_Advanced Mathematics_Complex number
Diploma_Semester-II_Advanced Mathematics_Complex number
 
Matrices
Matrices Matrices
Matrices
 

Similar to 3D Geometry Chapter 11 Maths

Chapter 10 vector_algebra
Chapter 10 vector_algebraChapter 10 vector_algebra
Chapter 10 vector_algebraShahrukh Javed
 
coordinate_geometry
 coordinate_geometry coordinate_geometry
coordinate_geometryKripi Mehra
 
Three Dimensional Geometry(Class 12)
Three Dimensional Geometry(Class 12)Three Dimensional Geometry(Class 12)
Three Dimensional Geometry(Class 12)TarunSingh389
 
Mathematics.pdf
Mathematics.pdfMathematics.pdf
Mathematics.pdfzaraa30
 
LOCUS AND BISECTOR OF THE ANGLE BETWEEN 2 LINES
LOCUS AND BISECTOR OF THE ANGLE BETWEEN 2 LINESLOCUS AND BISECTOR OF THE ANGLE BETWEEN 2 LINES
LOCUS AND BISECTOR OF THE ANGLE BETWEEN 2 LINESsumanmathews
 
CoordinateGeometry.ppt.pptx
CoordinateGeometry.ppt.pptxCoordinateGeometry.ppt.pptx
CoordinateGeometry.ppt.pptxAaryanLal
 
GCSE-StraightLines.pptx
GCSE-StraightLines.pptxGCSE-StraightLines.pptx
GCSE-StraightLines.pptxNourLahham2
 
HOW TO CALCULATE EQUATION OF A LINE(2018)
HOW TO CALCULATE EQUATION OF A LINE(2018)HOW TO CALCULATE EQUATION OF A LINE(2018)
HOW TO CALCULATE EQUATION OF A LINE(2018)sumanmathews
 
Chapter 6
Chapter 6Chapter 6
Chapter 6wzuri
 
Diploma-Semester-II_Advanced Mathematics_Unit-I
Diploma-Semester-II_Advanced Mathematics_Unit-IDiploma-Semester-II_Advanced Mathematics_Unit-I
Diploma-Semester-II_Advanced Mathematics_Unit-IRai University
 
Module 2 plane coordinate geometry
Module  2   plane coordinate geometryModule  2   plane coordinate geometry
Module 2 plane coordinate geometrydionesioable
 

Similar to 3D Geometry Chapter 11 Maths (20)

Chapter 10 vector_algebra
Chapter 10 vector_algebraChapter 10 vector_algebra
Chapter 10 vector_algebra
 
coordinate_geometry
 coordinate_geometry coordinate_geometry
coordinate_geometry
 
3D-PPt MODULE 1.pptx
3D-PPt MODULE 1.pptx3D-PPt MODULE 1.pptx
3D-PPt MODULE 1.pptx
 
Three Dimensional Geometry(Class 12)
Three Dimensional Geometry(Class 12)Three Dimensional Geometry(Class 12)
Three Dimensional Geometry(Class 12)
 
Mathematics.pdf
Mathematics.pdfMathematics.pdf
Mathematics.pdf
 
LOCUS AND BISECTOR OF THE ANGLE BETWEEN 2 LINES
LOCUS AND BISECTOR OF THE ANGLE BETWEEN 2 LINESLOCUS AND BISECTOR OF THE ANGLE BETWEEN 2 LINES
LOCUS AND BISECTOR OF THE ANGLE BETWEEN 2 LINES
 
CoordinateGeometry.ppt.pptx
CoordinateGeometry.ppt.pptxCoordinateGeometry.ppt.pptx
CoordinateGeometry.ppt.pptx
 
circles
 circles circles
circles
 
Straight line
Straight line Straight line
Straight line
 
GCSE-StraightLines.pptx
GCSE-StraightLines.pptxGCSE-StraightLines.pptx
GCSE-StraightLines.pptx
 
Vectors seminar
Vectors seminarVectors seminar
Vectors seminar
 
HOW TO CALCULATE EQUATION OF A LINE(2018)
HOW TO CALCULATE EQUATION OF A LINE(2018)HOW TO CALCULATE EQUATION OF A LINE(2018)
HOW TO CALCULATE EQUATION OF A LINE(2018)
 
3D Geometry Theory 3
3D Geometry Theory 33D Geometry Theory 3
3D Geometry Theory 3
 
Q addmaths 2011
Q addmaths 2011Q addmaths 2011
Q addmaths 2011
 
Chapter 6
Chapter 6Chapter 6
Chapter 6
 
1525 equations of lines in space
1525 equations of lines in space1525 equations of lines in space
1525 equations of lines in space
 
merged_document
merged_documentmerged_document
merged_document
 
Diploma-Semester-II_Advanced Mathematics_Unit-I
Diploma-Semester-II_Advanced Mathematics_Unit-IDiploma-Semester-II_Advanced Mathematics_Unit-I
Diploma-Semester-II_Advanced Mathematics_Unit-I
 
Module 2 plane coordinate geometry
Module  2   plane coordinate geometryModule  2   plane coordinate geometry
Module 2 plane coordinate geometry
 
Coordinate geometry
Coordinate geometryCoordinate geometry
Coordinate geometry
 

More from Shahrukh Javed

Chapter 12 linear_programming
Chapter 12 linear_programmingChapter 12 linear_programming
Chapter 12 linear_programmingShahrukh Javed
 
Chapter 9 differential_equations
Chapter 9 differential_equationsChapter 9 differential_equations
Chapter 9 differential_equationsShahrukh Javed
 
Chapter 8 application_of_integrals
Chapter 8 application_of_integralsChapter 8 application_of_integrals
Chapter 8 application_of_integralsShahrukh Javed
 
Chapter 6 application_of_derivatives
Chapter 6 application_of_derivativesChapter 6 application_of_derivatives
Chapter 6 application_of_derivativesShahrukh Javed
 
Chapter 5 continuity_and_differentiability
Chapter 5 continuity_and_differentiabilityChapter 5 continuity_and_differentiability
Chapter 5 continuity_and_differentiabilityShahrukh Javed
 
Chapter 4 determinants
Chapter 4 determinantsChapter 4 determinants
Chapter 4 determinantsShahrukh Javed
 
Chapter 2 inverse_trigonometric_functions
Chapter 2 inverse_trigonometric_functionsChapter 2 inverse_trigonometric_functions
Chapter 2 inverse_trigonometric_functionsShahrukh Javed
 
Chapter 1 relations_and_functions
Chapter 1 relations_and_functionsChapter 1 relations_and_functions
Chapter 1 relations_and_functionsShahrukh Javed
 
WHY CHILDREN ABSORBS MORE MICROWAVE RADIATION THAT ADULTS: THE CONSEQUENCES” ...
WHY CHILDREN ABSORBS MORE MICROWAVE RADIATION THAT ADULTS: THE CONSEQUENCES” ...WHY CHILDREN ABSORBS MORE MICROWAVE RADIATION THAT ADULTS: THE CONSEQUENCES” ...
WHY CHILDREN ABSORBS MORE MICROWAVE RADIATION THAT ADULTS: THE CONSEQUENCES” ...Shahrukh Javed
 
Universal asynchronous receiver_transmitter_uart_rs232
Universal asynchronous receiver_transmitter_uart_rs232Universal asynchronous receiver_transmitter_uart_rs232
Universal asynchronous receiver_transmitter_uart_rs232Shahrukh Javed
 
Imx53 uart- GUIDE BOOK
Imx53 uart- GUIDE BOOKImx53 uart- GUIDE BOOK
Imx53 uart- GUIDE BOOKShahrukh Javed
 
Heart rate counter explanation
Heart rate counter explanationHeart rate counter explanation
Heart rate counter explanationShahrukh Javed
 
Universal asynchronous receiver-transmitter UART Dsa project report
Universal asynchronous receiver-transmitter UART Dsa project reportUniversal asynchronous receiver-transmitter UART Dsa project report
Universal asynchronous receiver-transmitter UART Dsa project reportShahrukh Javed
 
Wireless power transmission
Wireless power transmissionWireless power transmission
Wireless power transmissionShahrukh Javed
 
Why children absorb more microwave radiation than adults the consequences
Why children absorb more microwave radiation than adults the consequencesWhy children absorb more microwave radiation than adults the consequences
Why children absorb more microwave radiation than adults the consequencesShahrukh Javed
 

More from Shahrukh Javed (20)

Class xii
Class xiiClass xii
Class xii
 
Chapter 12 linear_programming
Chapter 12 linear_programmingChapter 12 linear_programming
Chapter 12 linear_programming
 
Chapter 9 differential_equations
Chapter 9 differential_equationsChapter 9 differential_equations
Chapter 9 differential_equations
 
Chapter 8 application_of_integrals
Chapter 8 application_of_integralsChapter 8 application_of_integrals
Chapter 8 application_of_integrals
 
Chapter 7 integrals
Chapter 7 integralsChapter 7 integrals
Chapter 7 integrals
 
Chapter 6 application_of_derivatives
Chapter 6 application_of_derivativesChapter 6 application_of_derivatives
Chapter 6 application_of_derivatives
 
Chapter 5 continuity_and_differentiability
Chapter 5 continuity_and_differentiabilityChapter 5 continuity_and_differentiability
Chapter 5 continuity_and_differentiability
 
Chapter 4 determinants
Chapter 4 determinantsChapter 4 determinants
Chapter 4 determinants
 
Chapter 2 inverse_trigonometric_functions
Chapter 2 inverse_trigonometric_functionsChapter 2 inverse_trigonometric_functions
Chapter 2 inverse_trigonometric_functions
 
Chapter 1 relations_and_functions
Chapter 1 relations_and_functionsChapter 1 relations_and_functions
Chapter 1 relations_and_functions
 
WHY CHILDREN ABSORBS MORE MICROWAVE RADIATION THAT ADULTS: THE CONSEQUENCES” ...
WHY CHILDREN ABSORBS MORE MICROWAVE RADIATION THAT ADULTS: THE CONSEQUENCES” ...WHY CHILDREN ABSORBS MORE MICROWAVE RADIATION THAT ADULTS: THE CONSEQUENCES” ...
WHY CHILDREN ABSORBS MORE MICROWAVE RADIATION THAT ADULTS: THE CONSEQUENCES” ...
 
Universal asynchronous receiver_transmitter_uart_rs232
Universal asynchronous receiver_transmitter_uart_rs232Universal asynchronous receiver_transmitter_uart_rs232
Universal asynchronous receiver_transmitter_uart_rs232
 
Uart receiver
Uart receiverUart receiver
Uart receiver
 
Imx53 uart- GUIDE BOOK
Imx53 uart- GUIDE BOOKImx53 uart- GUIDE BOOK
Imx53 uart- GUIDE BOOK
 
Heart rate counter explanation
Heart rate counter explanationHeart rate counter explanation
Heart rate counter explanation
 
Universal asynchronous receiver-transmitter UART Dsa project report
Universal asynchronous receiver-transmitter UART Dsa project reportUniversal asynchronous receiver-transmitter UART Dsa project report
Universal asynchronous receiver-transmitter UART Dsa project report
 
BLACK BOX
BLACK  BOXBLACK  BOX
BLACK BOX
 
Wireless power transmission
Wireless power transmissionWireless power transmission
Wireless power transmission
 
Wireless charging
Wireless chargingWireless charging
Wireless charging
 
Why children absorb more microwave radiation than adults the consequences
Why children absorb more microwave radiation than adults the consequencesWhy children absorb more microwave radiation than adults the consequences
Why children absorb more microwave radiation than adults the consequences
 

Recently uploaded

History Class XII Ch. 3 Kinship, Caste and Class (1).pptx
History Class XII Ch. 3 Kinship, Caste and Class (1).pptxHistory Class XII Ch. 3 Kinship, Caste and Class (1).pptx
History Class XII Ch. 3 Kinship, Caste and Class (1).pptxsocialsciencegdgrohi
 
Employee wellbeing at the workplace.pptx
Employee wellbeing at the workplace.pptxEmployee wellbeing at the workplace.pptx
Employee wellbeing at the workplace.pptxNirmalaLoungPoorunde1
 
Pharmacognosy Flower 3. Compositae 2023.pdf
Pharmacognosy Flower 3. Compositae 2023.pdfPharmacognosy Flower 3. Compositae 2023.pdf
Pharmacognosy Flower 3. Compositae 2023.pdfMahmoud M. Sallam
 
Final demo Grade 9 for demo Plan dessert.pptx
Final demo Grade 9 for demo Plan dessert.pptxFinal demo Grade 9 for demo Plan dessert.pptx
Final demo Grade 9 for demo Plan dessert.pptxAvyJaneVismanos
 
Crayon Activity Handout For the Crayon A
Crayon Activity Handout For the Crayon ACrayon Activity Handout For the Crayon A
Crayon Activity Handout For the Crayon AUnboundStockton
 
भारत-रोम व्यापार.pptx, Indo-Roman Trade,
भारत-रोम व्यापार.pptx, Indo-Roman Trade,भारत-रोम व्यापार.pptx, Indo-Roman Trade,
भारत-रोम व्यापार.pptx, Indo-Roman Trade,Virag Sontakke
 
Introduction to AI in Higher Education_draft.pptx
Introduction to AI in Higher Education_draft.pptxIntroduction to AI in Higher Education_draft.pptx
Introduction to AI in Higher Education_draft.pptxpboyjonauth
 
Earth Day Presentation wow hello nice great
Earth Day Presentation wow hello nice greatEarth Day Presentation wow hello nice great
Earth Day Presentation wow hello nice greatYousafMalik24
 
Roles & Responsibilities in Pharmacovigilance
Roles & Responsibilities in PharmacovigilanceRoles & Responsibilities in Pharmacovigilance
Roles & Responsibilities in PharmacovigilanceSamikshaHamane
 
Meghan Sutherland In Media Res Media Component
Meghan Sutherland In Media Res Media ComponentMeghan Sutherland In Media Res Media Component
Meghan Sutherland In Media Res Media ComponentInMediaRes1
 
Alper Gobel In Media Res Media Component
Alper Gobel In Media Res Media ComponentAlper Gobel In Media Res Media Component
Alper Gobel In Media Res Media ComponentInMediaRes1
 
18-04-UA_REPORT_MEDIALITERAСY_INDEX-DM_23-1-final-eng.pdf
18-04-UA_REPORT_MEDIALITERAСY_INDEX-DM_23-1-final-eng.pdf18-04-UA_REPORT_MEDIALITERAСY_INDEX-DM_23-1-final-eng.pdf
18-04-UA_REPORT_MEDIALITERAСY_INDEX-DM_23-1-final-eng.pdfssuser54595a
 
Interactive Powerpoint_How to Master effective communication
Interactive Powerpoint_How to Master effective communicationInteractive Powerpoint_How to Master effective communication
Interactive Powerpoint_How to Master effective communicationnomboosow
 
How to Make a Pirate ship Primary Education.pptx
How to Make a Pirate ship Primary Education.pptxHow to Make a Pirate ship Primary Education.pptx
How to Make a Pirate ship Primary Education.pptxmanuelaromero2013
 
CELL CYCLE Division Science 8 quarter IV.pptx
CELL CYCLE Division Science 8 quarter IV.pptxCELL CYCLE Division Science 8 quarter IV.pptx
CELL CYCLE Division Science 8 quarter IV.pptxJiesonDelaCerna
 
MARGINALIZATION (Different learners in Marginalized Group
MARGINALIZATION (Different learners in Marginalized GroupMARGINALIZATION (Different learners in Marginalized Group
MARGINALIZATION (Different learners in Marginalized GroupJonathanParaisoCruz
 
DATA STRUCTURE AND ALGORITHM for beginners
DATA STRUCTURE AND ALGORITHM for beginnersDATA STRUCTURE AND ALGORITHM for beginners
DATA STRUCTURE AND ALGORITHM for beginnersSabitha Banu
 
KSHARA STURA .pptx---KSHARA KARMA THERAPY (CAUSTIC THERAPY)————IMP.OF KSHARA ...
KSHARA STURA .pptx---KSHARA KARMA THERAPY (CAUSTIC THERAPY)————IMP.OF KSHARA ...KSHARA STURA .pptx---KSHARA KARMA THERAPY (CAUSTIC THERAPY)————IMP.OF KSHARA ...
KSHARA STURA .pptx---KSHARA KARMA THERAPY (CAUSTIC THERAPY)————IMP.OF KSHARA ...M56BOOKSTORE PRODUCT/SERVICE
 
Incoming and Outgoing Shipments in 1 STEP Using Odoo 17
Incoming and Outgoing Shipments in 1 STEP Using Odoo 17Incoming and Outgoing Shipments in 1 STEP Using Odoo 17
Incoming and Outgoing Shipments in 1 STEP Using Odoo 17Celine George
 

Recently uploaded (20)

History Class XII Ch. 3 Kinship, Caste and Class (1).pptx
History Class XII Ch. 3 Kinship, Caste and Class (1).pptxHistory Class XII Ch. 3 Kinship, Caste and Class (1).pptx
History Class XII Ch. 3 Kinship, Caste and Class (1).pptx
 
Employee wellbeing at the workplace.pptx
Employee wellbeing at the workplace.pptxEmployee wellbeing at the workplace.pptx
Employee wellbeing at the workplace.pptx
 
Pharmacognosy Flower 3. Compositae 2023.pdf
Pharmacognosy Flower 3. Compositae 2023.pdfPharmacognosy Flower 3. Compositae 2023.pdf
Pharmacognosy Flower 3. Compositae 2023.pdf
 
Final demo Grade 9 for demo Plan dessert.pptx
Final demo Grade 9 for demo Plan dessert.pptxFinal demo Grade 9 for demo Plan dessert.pptx
Final demo Grade 9 for demo Plan dessert.pptx
 
Crayon Activity Handout For the Crayon A
Crayon Activity Handout For the Crayon ACrayon Activity Handout For the Crayon A
Crayon Activity Handout For the Crayon A
 
भारत-रोम व्यापार.pptx, Indo-Roman Trade,
भारत-रोम व्यापार.pptx, Indo-Roman Trade,भारत-रोम व्यापार.pptx, Indo-Roman Trade,
भारत-रोम व्यापार.pptx, Indo-Roman Trade,
 
Introduction to AI in Higher Education_draft.pptx
Introduction to AI in Higher Education_draft.pptxIntroduction to AI in Higher Education_draft.pptx
Introduction to AI in Higher Education_draft.pptx
 
Earth Day Presentation wow hello nice great
Earth Day Presentation wow hello nice greatEarth Day Presentation wow hello nice great
Earth Day Presentation wow hello nice great
 
Roles & Responsibilities in Pharmacovigilance
Roles & Responsibilities in PharmacovigilanceRoles & Responsibilities in Pharmacovigilance
Roles & Responsibilities in Pharmacovigilance
 
Meghan Sutherland In Media Res Media Component
Meghan Sutherland In Media Res Media ComponentMeghan Sutherland In Media Res Media Component
Meghan Sutherland In Media Res Media Component
 
Alper Gobel In Media Res Media Component
Alper Gobel In Media Res Media ComponentAlper Gobel In Media Res Media Component
Alper Gobel In Media Res Media Component
 
18-04-UA_REPORT_MEDIALITERAСY_INDEX-DM_23-1-final-eng.pdf
18-04-UA_REPORT_MEDIALITERAСY_INDEX-DM_23-1-final-eng.pdf18-04-UA_REPORT_MEDIALITERAСY_INDEX-DM_23-1-final-eng.pdf
18-04-UA_REPORT_MEDIALITERAСY_INDEX-DM_23-1-final-eng.pdf
 
Interactive Powerpoint_How to Master effective communication
Interactive Powerpoint_How to Master effective communicationInteractive Powerpoint_How to Master effective communication
Interactive Powerpoint_How to Master effective communication
 
How to Make a Pirate ship Primary Education.pptx
How to Make a Pirate ship Primary Education.pptxHow to Make a Pirate ship Primary Education.pptx
How to Make a Pirate ship Primary Education.pptx
 
CELL CYCLE Division Science 8 quarter IV.pptx
CELL CYCLE Division Science 8 quarter IV.pptxCELL CYCLE Division Science 8 quarter IV.pptx
CELL CYCLE Division Science 8 quarter IV.pptx
 
MARGINALIZATION (Different learners in Marginalized Group
MARGINALIZATION (Different learners in Marginalized GroupMARGINALIZATION (Different learners in Marginalized Group
MARGINALIZATION (Different learners in Marginalized Group
 
DATA STRUCTURE AND ALGORITHM for beginners
DATA STRUCTURE AND ALGORITHM for beginnersDATA STRUCTURE AND ALGORITHM for beginners
DATA STRUCTURE AND ALGORITHM for beginners
 
KSHARA STURA .pptx---KSHARA KARMA THERAPY (CAUSTIC THERAPY)————IMP.OF KSHARA ...
KSHARA STURA .pptx---KSHARA KARMA THERAPY (CAUSTIC THERAPY)————IMP.OF KSHARA ...KSHARA STURA .pptx---KSHARA KARMA THERAPY (CAUSTIC THERAPY)————IMP.OF KSHARA ...
KSHARA STURA .pptx---KSHARA KARMA THERAPY (CAUSTIC THERAPY)————IMP.OF KSHARA ...
 
Incoming and Outgoing Shipments in 1 STEP Using Odoo 17
Incoming and Outgoing Shipments in 1 STEP Using Odoo 17Incoming and Outgoing Shipments in 1 STEP Using Odoo 17
Incoming and Outgoing Shipments in 1 STEP Using Odoo 17
 
OS-operating systems- ch04 (Threads) ...
OS-operating systems- ch04 (Threads) ...OS-operating systems- ch04 (Threads) ...
OS-operating systems- ch04 (Threads) ...
 

3D Geometry Chapter 11 Maths

  • 1. Class XII Chapter 11 – Three Dimensional Geometry Maths Page 1 of 58 Website: www.vidhyarjan.com Email: contact@vidhyarjan.com Mobile: 9999 249717 Head Office: 1/3-H-A-2, Street # 6, East Azad Nagar, Delhi-110051 (One Km from ‘Welcome’ Metro Station) Exercise 11.1 Question 1: If a line makes angles 90°, 135°, 45° with x, y and z-axes respectively, find its direction cosines. Answer Let direction cosines of the line be l, m, and n. Therefore, the direction cosines of the line are Question 2: Find the direction cosines of a line which makes equal angles with the coordinate axes. Answer Let the direction cosines of the line make an angle α with each of the coordinate axes. ∴ l = cos α, m = cos α, n = cos α
  • 2. Class XII Chapter 11 – Three Dimensional Geometry Maths Page 2 of 58 Website: www.vidhyarjan.com Email: contact@vidhyarjan.com Mobile: 9999 249717 Head Office: 1/3-H-A-2, Street # 6, East Azad Nagar, Delhi-110051 (One Km from ‘Welcome’ Metro Station) Thus, the direction cosines of the line, which is equally inclined to the coordinate axes, are Question 3: If a line has the direction ratios −18, 12, −4, then what are its direction cosines? Answer If a line has direction ratios of −18, 12, and −4, then its direction cosines are Thus, the direction cosines are . Question 4: Show that the points (2, 3, 4), (−1, −2, 1), (5, 8, 7) are collinear. Answer The given points are A (2, 3, 4), B (− 1, − 2, 1), and C (5, 8, 7). It is known that the direction ratios of line joining the points, (x1, y1, z1) and (x2, y2, z2), are given by, x2 − x1, y2 − y1, and z2 − z1. The direction ratios of AB are (−1 − 2), (−2 − 3), and (1 − 4) i.e., −3, −5, and −3. The direction ratios of BC are (5 − (− 1)), (8 − (− 2)), and (7 − 1) i.e., 6, 10, and 6. It can be seen that the direction ratios of BC are −2 times that of AB i.e., they are proportional. Therefore, AB is parallel to BC. Since point B is common to both AB and BC, points A, B, and C are collinear.
  • 3. Class XII Chapter 11 – Three Dimensional Geometry Maths Page 3 of 58 Website: www.vidhyarjan.com Email: contact@vidhyarjan.com Mobile: 9999 249717 Head Office: 1/3-H-A-2, Street # 6, East Azad Nagar, Delhi-110051 (One Km from ‘Welcome’ Metro Station) Question 5: Find the direction cosines of the sides of the triangle whose vertices are (3, 5, − 4), (− 1, 1, 2) and (− 5, − 5, − 2) Answer The vertices of ∆ABC are A (3, 5, −4), B (−1, 1, 2), and C (−5, −5, −2). The direction ratios of side AB are (−1 − 3), (1 − 5), and (2 − (−4)) i.e., −4, −4, and 6. Therefore, the direction cosines of AB are The direction ratios of BC are (−5 − (−1)), (−5 − 1), and (−2 − 2) i.e., −4, −6, and −4. Therefore, the direction cosines of BC are The direction ratios of CA are (−5 − 3), (−5 − 5), and (−2 − (−4)) i.e., −8, −10, and 2. Therefore, the direction cosines of AC are
  • 4. Class XII Chapter 11 – Three Dimensional Geometry Maths Page 4 of 58 Website: www.vidhyarjan.com Email: contact@vidhyarjan.com Mobile: 9999 249717 Head Office: 1/3-H-A-2, Street # 6, East Azad Nagar, Delhi-110051 (One Km from ‘Welcome’ Metro Station)
  • 5. Class XII Chapter 11 – Three Dimensional Geometry Maths Page 5 of 58 Website: www.vidhyarjan.com Email: contact@vidhyarjan.com Mobile: 9999 249717 Head Office: 1/3-H-A-2, Street # 6, East Azad Nagar, Delhi-110051 (One Km from ‘Welcome’ Metro Station) Exercise 11.2 Question 1: Show that the three lines with direction cosines are mutually perpendicular. Answer Two lines with direction cosines, l1, m1, n1 and l2, m2, n2, are perpendicular to each other, if l1l2 + m1m2 + n1n2 = 0 (i) For the lines with direction cosines, and , we obtain Therefore, the lines are perpendicular. (ii) For the lines with direction cosines, and , we obtain Therefore, the lines are perpendicular. (iii) For the lines with direction cosines, and , we obtain
  • 6. Class XII Chapter 11 – Three Dimensional Geometry Maths Page 6 of 58 Website: www.vidhyarjan.com Email: contact@vidhyarjan.com Mobile: 9999 249717 Head Office: 1/3-H-A-2, Street # 6, East Azad Nagar, Delhi-110051 (One Km from ‘Welcome’ Metro Station) Therefore, the lines are perpendicular. Thus, all the lines are mutually perpendicular. Question 2: Show that the line through the points (1, −1, 2) (3, 4, −2) is perpendicular to the line through the points (0, 3, 2) and (3, 5, 6). Answer Let AB be the line joining the points, (1, −1, 2) and (3, 4, − 2), and CD be the line joining the points, (0, 3, 2) and (3, 5, 6). The direction ratios, a1, b1, c1, of AB are (3 − 1), (4 − (−1)), and (−2 − 2) i.e., 2, 5, and −4. The direction ratios, a2, b2, c2, of CD are (3 − 0), (5 − 3), and (6 −2) i.e., 3, 2, and 4. AB and CD will be perpendicular to each other, if a1a2 + b1b2+ c1c2 = 0 a1a2 + b1b2+ c1c2 = 2 × 3 + 5 × 2 + (− 4) × 4 = 6 + 10 − 16 = 0 Therefore, AB and CD are perpendicular to each other. Question 3: Show that the line through the points (4, 7, 8) (2, 3, 4) is parallel to the line through the points (−1, −2, 1), (1, 2, 5). Answer Let AB be the line through the points, (4, 7, 8) and (2, 3, 4), and CD be the line through the points, (−1, −2, 1) and (1, 2, 5). The directions ratios, a1, b1, c1, of AB are (2 − 4), (3 − 7), and (4 − 8) i.e., −2, −4, and −4. The direction ratios, a2, b2, c2, of CD are (1 − (−1)), (2 − (−2)), and (5 − 1) i.e., 2, 4, and 4.
  • 7. Class XII Chapter 11 – Three Dimensional Geometry Maths Page 7 of 58 Website: www.vidhyarjan.com Email: contact@vidhyarjan.com Mobile: 9999 249717 Head Office: 1/3-H-A-2, Street # 6, East Azad Nagar, Delhi-110051 (One Km from ‘Welcome’ Metro Station) AB will be parallel to CD, if Thus, AB is parallel to CD. Question 4: Find the equation of the line which passes through the point (1, 2, 3) and is parallel to the vector . Answer It is given that the line passes through the point A (1, 2, 3). Therefore, the position vector through A is It is known that the line which passes through point A and parallel to is given by is a constant. This is the required equation of the line. Question 5: Find the equation of the line in vector and in Cartesian form that passes through the point with position vector and is in the direction . Answer It is given that the line passes through the point with position vector
  • 8. Class XII Chapter 11 – Three Dimensional Geometry Maths Page 8 of 58 Website: www.vidhyarjan.com Email: contact@vidhyarjan.com Mobile: 9999 249717 Head Office: 1/3-H-A-2, Street # 6, East Azad Nagar, Delhi-110051 (One Km from ‘Welcome’ Metro Station) It is known that a line through a point with position vector and parallel to is given by the equation, This is the required equation of the line in vector form. Eliminating λ, we obtain the Cartesian form equation as This is the required equation of the given line in Cartesian form. Question 6: Find the Cartesian equation of the line which passes through the point (−2, 4, −5) and parallel to the line given by Answer It is given that the line passes through the point (−2, 4, −5) and is parallel to The direction ratios of the line, , are 3, 5, and 6. The required line is parallel to Therefore, its direction ratios are 3k, 5k, and 6k, where k ≠ 0 It is known that the equation of the line through the point (x1, y1, z1) and with direction ratios, a, b, c, is given by
  • 9. Class XII Chapter 11 – Three Dimensional Geometry Maths Page 9 of 58 Website: www.vidhyarjan.com Email: contact@vidhyarjan.com Mobile: 9999 249717 Head Office: 1/3-H-A-2, Street # 6, East Azad Nagar, Delhi-110051 (One Km from ‘Welcome’ Metro Station) Therefore the equation of the required line is Question 7: The Cartesian equation of a line is . Write its vector form. Answer The Cartesian equation of the line is The given line passes through the point (5, −4, 6). The position vector of this point is Also, the direction ratios of the given line are 3, 7, and 2. This means that the line is in the direction of vector, It is known that the line through position vector and in the direction of the vector is given by the equation, This is the required equation of the given line in vector form. Question 8: Find the vector and the Cartesian equations of the lines that pass through the origin and (5, −2, 3). Answer The required line passes through the origin. Therefore, its position vector is given by, The direction ratios of the line through origin and (5, −2, 3) are (5 − 0) = 5, (−2 − 0) = −2, (3 − 0) = 3
  • 10. Class XII Chapter 11 – Three Dimensional Geometry Maths Page 10 of 58 Website: www.vidhyarjan.com Email: contact@vidhyarjan.com Mobile: 9999 249717 Head Office: 1/3-H-A-2, Street # 6, East Azad Nagar, Delhi-110051 (One Km from ‘Welcome’ Metro Station) The line is parallel to the vector given by the equation, The equation of the line in vector form through a point with position vector and parallel to is, The equation of the line through the point (x1, y1, z1) and direction ratios a, b, c is given by, Therefore, the equation of the required line in the Cartesian form is Question 9: Find the vector and the Cartesian equations of the line that passes through the points (3, −2, −5), (3, −2, 6). Answer Let the line passing through the points, P (3, −2, −5) and Q (3, −2, 6), be PQ. Since PQ passes through P (3, −2, −5), its position vector is given by, The direction ratios of PQ are given by, (3 − 3) = 0, (−2 + 2) = 0, (6 + 5) = 11 The equation of the vector in the direction of PQ is The equation of PQ in vector form is given by, The equation of PQ in Cartesian form is
  • 11. Class XII Chapter 11 – Three Dimensional Geometry Maths Page 11 of 58 Website: www.vidhyarjan.com Email: contact@vidhyarjan.com Mobile: 9999 249717 Head Office: 1/3-H-A-2, Street # 6, East Azad Nagar, Delhi-110051 (One Km from ‘Welcome’ Metro Station) i.e., Question 10: Find the angle between the following pairs of lines: (i) (ii) and Answer (i) Let Q be the angle between the given lines. The angle between the given pairs of lines is given by, The given lines are parallel to the vectors, and , respectively.
  • 12. Class XII Chapter 11 – Three Dimensional Geometry Maths Page 12 of 58 Website: www.vidhyarjan.com Email: contact@vidhyarjan.com Mobile: 9999 249717 Head Office: 1/3-H-A-2, Street # 6, East Azad Nagar, Delhi-110051 (One Km from ‘Welcome’ Metro Station) (ii) The given lines are parallel to the vectors, and , respectively. Question 11: Find the angle between the following pairs of lines: (i) (ii) i. Answer
  • 13. Class XII Chapter 11 – Three Dimensional Geometry Maths Page 13 of 58 Website: www.vidhyarjan.com Email: contact@vidhyarjan.com Mobile: 9999 249717 Head Office: 1/3-H-A-2, Street # 6, East Azad Nagar, Delhi-110051 (One Km from ‘Welcome’ Metro Station) ii. Let and be the vectors parallel to the pair of lines, , respectively. and The angle, Q, between the given pair of lines is given by the relation, (ii) Let be the vectors parallel to the given pair of lines, and , respectively.
  • 14. Class XII Chapter 11 – Three Dimensional Geometry Maths Page 14 of 58 Website: www.vidhyarjan.com Email: contact@vidhyarjan.com Mobile: 9999 249717 Head Office: 1/3-H-A-2, Street # 6, East Azad Nagar, Delhi-110051 (One Km from ‘Welcome’ Metro Station) If Q is the angle between the given pair of lines, then Question 12: Find the values of p so the line and are at right angles. Answer The given equations can be written in the standard form as and The direction ratios of the lines are −3, , 2 and respectively. Two lines with direction ratios, a1, b1, c1 and a2, b2, c2, are perpendicular to each other, if a1a2 + b1 b2 + c1c2 = 0
  • 15. Class XII Chapter 11 – Three Dimensional Geometry Maths Page 15 of 58 Website: www.vidhyarjan.com Email: contact@vidhyarjan.com Mobile: 9999 249717 Head Office: 1/3-H-A-2, Street # 6, East Azad Nagar, Delhi-110051 (One Km from ‘Welcome’ Metro Station) Thus, the value of p is . Question 13: Show that the lines and are perpendicular to each other. Answer The equations of the given lines are and The direction ratios of the given lines are 7, −5, 1 and 1, 2, 3 respectively. Two lines with direction ratios, a1, b1, c1 and a2, b2, c2, are perpendicular to each other, if a1a2 + b1 b2 + c1c2 = 0 ∴ 7 × 1 + (−5) × 2 + 1 × 3 = 7 − 10 + 3 = 0 Therefore, the given lines are perpendicular to each other. Question 14: Find the shortest distance between the lines
  • 16. Class XII Chapter 11 – Three Dimensional Geometry Maths Page 16 of 58 Website: www.vidhyarjan.com Email: contact@vidhyarjan.com Mobile: 9999 249717 Head Office: 1/3-H-A-2, Street # 6, East Azad Nagar, Delhi-110051 (One Km from ‘Welcome’ Metro Station) Answer The equations of the given lines are It is known that the shortest distance between the lines, and , is given by, Comparing the given equations, we obtain Substituting all the values in equation (1), we obtain
  • 17. Class XII Chapter 11 – Three Dimensional Geometry Maths Page 17 of 58 Website: www.vidhyarjan.com Email: contact@vidhyarjan.com Mobile: 9999 249717 Head Office: 1/3-H-A-2, Street # 6, East Azad Nagar, Delhi-110051 (One Km from ‘Welcome’ Metro Station) Therefore, the shortest distance between the two lines is units. Question 15: Find the shortest distance between the lines and Answer The given lines are and It is known that the shortest distance between the two lines, , is given by, Comparing the given equations, we obtain
  • 18. Class XII Chapter 11 – Three Dimensional Geometry Maths Page 18 of 58 Website: www.vidhyarjan.com Email: contact@vidhyarjan.com Mobile: 9999 249717 Head Office: 1/3-H-A-2, Street # 6, East Azad Nagar, Delhi-110051 (One Km from ‘Welcome’ Metro Station) Substituting all the values in equation (1), we obtain Since distance is always non-negative, the distance between the given lines is units. Question 16: Find the shortest distance between the lines whose vector equations are Answer The given lines are and It is known that the shortest distance between the lines, and , is given by,
  • 19. Class XII Chapter 11 – Three Dimensional Geometry Maths Page 19 of 58 Website: www.vidhyarjan.com Email: contact@vidhyarjan.com Mobile: 9999 249717 Head Office: 1/3-H-A-2, Street # 6, East Azad Nagar, Delhi-110051 (One Km from ‘Welcome’ Metro Station) Comparing the given equations with and , we obtain Substituting all the values in equation (1), we obtain Therefore, the shortest distance between the two given lines is units. Question 17: Find the shortest distance between the lines whose vector equations are Answer The given lines are
  • 20. Class XII Chapter 11 – Three Dimensional Geometry Maths Page 20 of 58 Website: www.vidhyarjan.com Email: contact@vidhyarjan.com Mobile: 9999 249717 Head Office: 1/3-H-A-2, Street # 6, East Azad Nagar, Delhi-110051 (One Km from ‘Welcome’ Metro Station) It is known that the shortest distance between the lines, and , is given by, For the given equations, Substituting all the values in equation (3), we obtain Therefore, the shortest distance between the lines is units.
  • 21. Class XII Chapter 11 – Three Dimensional Geometry Maths Page 21 of 58 Website: www.vidhyarjan.com Email: contact@vidhyarjan.com Mobile: 9999 249717 Head Office: 1/3-H-A-2, Street # 6, East Azad Nagar, Delhi-110051 (One Km from ‘Welcome’ Metro Station) Exercise 11.3 Question 1: In each of the following cases, determine the direction cosines of the normal to the plane and the distance from the origin. (a)z = 2 (b) (c) (d)5y + 8 = 0 Answer (a) The equation of the plane is z = 2 or 0x + 0y + z = 2 … (1) The direction ratios of normal are 0, 0, and 1. ∴ Dividing both sides of equation (1) by 1, we obtain This is of the form lx + my + nz = d, where l, m, n are the direction cosines of normal to the plane and d is the distance of the perpendicular drawn from the origin. Therefore, the direction cosines are 0, 0, and 1 and the distance of the plane from the origin is 2 units. (b) x + y + z = 1 … (1) The direction ratios of normal are 1, 1, and 1. ∴ Dividing both sides of equation (1) by , we obtain
  • 22. Class XII Chapter 11 – Three Dimensional Geometry Maths Page 22 of 58 Website: www.vidhyarjan.com Email: contact@vidhyarjan.com Mobile: 9999 249717 Head Office: 1/3-H-A-2, Street # 6, East Azad Nagar, Delhi-110051 (One Km from ‘Welcome’ Metro Station) This equation is of the form lx + my + nz = d, where l, m, n are the direction cosines of normal to the plane and d is the distance of normal from the origin. Therefore, the direction cosines of the normal are and the distance of normal from the origin is units. (c) 2x + 3y − z = 5 … (1) The direction ratios of normal are 2, 3, and −1. Dividing both sides of equation (1) by , we obtain This equation is of the form lx + my + nz = d, where l, m, n are the direction cosines of normal to the plane and d is the distance of normal from the origin. Therefore, the direction cosines of the normal to the plane are and the distance of normal from the origin is units. (d) 5y + 8 = 0 ⇒ 0x − 5y + 0z = 8 … (1) The direction ratios of normal are 0, −5, and 0. Dividing both sides of equation (1) by 5, we obtain
  • 23. Class XII Chapter 11 – Three Dimensional Geometry Maths Page 23 of 58 Website: www.vidhyarjan.com Email: contact@vidhyarjan.com Mobile: 9999 249717 Head Office: 1/3-H-A-2, Street # 6, East Azad Nagar, Delhi-110051 (One Km from ‘Welcome’ Metro Station) This equation is of the form lx + my + nz = d, where l, m, n are the direction cosines of normal to the plane and d is the distance of normal from the origin. Therefore, the direction cosines of the normal to the plane are 0, −1, and 0 and the distance of normal from the origin is units. Question 2: Find the vector equation of a plane which is at a distance of 7 units from the origin and normal to the vector . Answer The normal vector is, It is known that the equation of the plane with position vector is given by, This is the vector equation of the required plane. Question 3: Find the Cartesian equation of the following planes: (a) (b) (c) Answer (a) It is given that equation of the plane is For any arbitrary point P (x, y, z) on the plane, position vector is given by,
  • 24. Class XII Chapter 11 – Three Dimensional Geometry Maths Page 24 of 58 Website: www.vidhyarjan.com Email: contact@vidhyarjan.com Mobile: 9999 249717 Head Office: 1/3-H-A-2, Street # 6, East Azad Nagar, Delhi-110051 (One Km from ‘Welcome’ Metro Station) Substituting the value of in equation (1), we obtain This is the Cartesian equation of the plane. (b) For any arbitrary point P (x, y, z) on the plane, position vector is given by, Substituting the value of in equation (1), we obtain This is the Cartesian equation of the plane. (c) For any arbitrary point P (x, y, z) on the plane, position vector is given by, Substituting the value of in equation (1), we obtain This is the Cartesian equation of the given plane. Question 4: In the following cases, find the coordinates of the foot of the perpendicular drawn from the origin. (a) (b) (c) (d) Answer (a) Let the coordinates of the foot of perpendicular P from the origin to the plane be (x1, y1, z1).
  • 25. Class XII Chapter 11 – Three Dimensional Geometry Maths Page 25 of 58 Website: www.vidhyarjan.com Email: contact@vidhyarjan.com Mobile: 9999 249717 Head Office: 1/3-H-A-2, Street # 6, East Azad Nagar, Delhi-110051 (One Km from ‘Welcome’ Metro Station) 2x + 3y + 4z − 12 = 0 ⇒ 2x + 3y + 4z = 12 … (1) The direction ratios of normal are 2, 3, and 4. Dividing both sides of equation (1) by , we obtain This equation is of the form lx + my + nz = d, where l, m, n are the direction cosines of normal to the plane and d is the distance of normal from the origin. The coordinates of the foot of the perpendicular are given by (ld, md, nd). Therefore, the coordinates of the foot of the perpendicular are (b) Let the coordinates of the foot of perpendicular P from the origin to the plane be (x1, y1, z1). ⇒ … (1) The direction ratios of the normal are 0, 3, and 4. Dividing both sides of equation (1) by 5, we obtain
  • 26. Class XII Chapter 11 – Three Dimensional Geometry Maths Page 26 of 58 Website: www.vidhyarjan.com Email: contact@vidhyarjan.com Mobile: 9999 249717 Head Office: 1/3-H-A-2, Street # 6, East Azad Nagar, Delhi-110051 (One Km from ‘Welcome’ Metro Station) This equation is of the form lx + my + nz = d, where l, m, n are the direction cosines of normal to the plane and d is the distance of normal from the origin. The coordinates of the foot of the perpendicular are given by (ld, md, nd). Therefore, the coordinates of the foot of the perpendicular are (c) Let the coordinates of the foot of perpendicular P from the origin to the plane be (x1, y1, z1). … (1) The direction ratios of the normal are 1, 1, and 1. Dividing both sides of equation (1) by , we obtain This equation is of the form lx + my + nz = d, where l, m, n are the direction cosines of normal to the plane and d is the distance of normal from the origin. The coordinates of the foot of the perpendicular are given by (ld, md, nd). Therefore, the coordinates of the foot of the perpendicular are (d) Let the coordinates of the foot of perpendicular P from the origin to the plane be (x1, y1, z1). ⇒ 0x − 5y + 0z = 8 … (1) The direction ratios of the normal are 0, −5, and 0.
  • 27. Class XII Chapter 11 – Three Dimensional Geometry Maths Page 27 of 58 Website: www.vidhyarjan.com Email: contact@vidhyarjan.com Mobile: 9999 249717 Head Office: 1/3-H-A-2, Street # 6, East Azad Nagar, Delhi-110051 (One Km from ‘Welcome’ Metro Station) Dividing both sides of equation (1) by 5, we obtain This equation is of the form lx + my + nz = d, where l, m, n are the direction cosines of normal to the plane and d is the distance of normal from the origin. The coordinates of the foot of the perpendicular are given by (ld, md, nd). Therefore, the coordinates of the foot of the perpendicular are Question 5: Find the vector and Cartesian equation of the planes (a) that passes through the point (1, 0, −2) and the normal to the plane is . (b) that passes through the point (1, 4, 6) and the normal vector to the plane is . Answer (a) The position vector of point (1, 0, −2) is The normal vector perpendicular to the plane is The vector equation of the plane is given by, is the position vector of any point P (x, y, z) in the plane. Therefore, equation (1) becomes
  • 28. Class XII Chapter 11 – Three Dimensional Geometry Maths Page 28 of 58 Website: www.vidhyarjan.com Email: contact@vidhyarjan.com Mobile: 9999 249717 Head Office: 1/3-H-A-2, Street # 6, East Azad Nagar, Delhi-110051 (One Km from ‘Welcome’ Metro Station) This is the Cartesian equation of the required plane. (b) The position vector of the point (1, 4, 6) is The normal vector perpendicular to the plane is The vector equation of the plane is given by, is the position vector of any point P (x, y, z) in the plane. Therefore, equation (1) becomes This is the Cartesian equation of the required plane. Question 6: Find the equations of the planes that passes through three points. (a) (1, 1, −1), (6, 4, −5), (−4, −2, 3) (b) (1, 1, 0), (1, 2, 1), (−2, 2, −1) Answer (a) The given points are A (1, 1, −1), B (6, 4, −5), and C (−4, −2, 3).
  • 29. Class XII Chapter 11 – Three Dimensional Geometry Maths Page 29 of 58 Website: www.vidhyarjan.com Email: contact@vidhyarjan.com Mobile: 9999 249717 Head Office: 1/3-H-A-2, Street # 6, East Azad Nagar, Delhi-110051 (One Km from ‘Welcome’ Metro Station) Since A, B, C are collinear points, there will be infinite number of planes passing through the given points. (b) The given points are A (1, 1, 0), B (1, 2, 1), and C (−2, 2, −1). Therefore, a plane will pass through the points A, B, and C. It is known that the equation of the plane through the points, , and , is This is the Cartesian equation of the required plane. Question 7: Find the intercepts cut off by the plane Answer
  • 30. Class XII Chapter 11 – Three Dimensional Geometry Maths Page 30 of 58 Website: www.vidhyarjan.com Email: contact@vidhyarjan.com Mobile: 9999 249717 Head Office: 1/3-H-A-2, Street # 6, East Azad Nagar, Delhi-110051 (One Km from ‘Welcome’ Metro Station) Dividing both sides of equation (1) by 5, we obtain It is known that the equation of a plane in intercept form is , where a, b, c are the intercepts cut off by the plane at x, y, and z axes respectively. Therefore, for the given equation, Thus, the intercepts cut off by the plane are . Question 8: Find the equation of the plane with intercept 3 on the y-axis and parallel to ZOX plane. Answer The equation of the plane ZOX is y = 0 Any plane parallel to it is of the form, y = a Since the y-intercept of the plane is 3, ∴ a = 3 Thus, the equation of the required plane is y = 3 Question 9: Find the equation of the plane through the intersection of the planes and and the point (2, 2, 1) Answer
  • 31. Class XII Chapter 11 – Three Dimensional Geometry Maths Page 31 of 58 Website: www.vidhyarjan.com Email: contact@vidhyarjan.com Mobile: 9999 249717 Head Office: 1/3-H-A-2, Street # 6, East Azad Nagar, Delhi-110051 (One Km from ‘Welcome’ Metro Station) The equation of any plane through the intersection of the planes, 3x − y + 2z − 4 = 0 and x + y + z − 2 = 0, is The plane passes through the point (2, 2, 1). Therefore, this point will satisfy equation (1). Substituting in equation (1), we obtain This is the required equation of the plane. Question 10: Find the vector equation of the plane passing through the intersection of the planes and through the point (2, 1, 3) Answer The equations of the planes are The equation of any plane through the intersection of the planes given in equations (1) and (2) is given by, , where
  • 32. Class XII Chapter 11 – Three Dimensional Geometry Maths Page 32 of 58 Website: www.vidhyarjan.com Email: contact@vidhyarjan.com Mobile: 9999 249717 Head Office: 1/3-H-A-2, Street # 6, East Azad Nagar, Delhi-110051 (One Km from ‘Welcome’ Metro Station) The plane passes through the point (2, 1, 3). Therefore, its position vector is given by, Substituting in equation (3), we obtain Substituting in equation (3), we obtain This is the vector equation of the required plane. Question 11: Find the equation of the plane through the line of intersection of the planes and which is perpendicular to the plane Answer The equation of the plane through the intersection of the planes, and , is The direction ratios, a1, b1, c1, of this plane are (2λ + 1), (3λ + 1), and (4λ + 1). The plane in equation (1) is perpendicular to
  • 33. Class XII Chapter 11 – Three Dimensional Geometry Maths Page 33 of 58 Website: www.vidhyarjan.com Email: contact@vidhyarjan.com Mobile: 9999 249717 Head Office: 1/3-H-A-2, Street # 6, East Azad Nagar, Delhi-110051 (One Km from ‘Welcome’ Metro Station) Its direction ratios, a2, b2, c2, are 1, −1, and 1. Since the planes are perpendicular, Substituting in equation (1), we obtain This is the required equation of the plane. Question 12: Find the angle between the planes whose vector equations are and . Answer The equations of the given planes are and It is known that if and are normal to the planes, and , then the angle between them, Q, is given by, Here,
  • 34. Class XII Chapter 11 – Three Dimensional Geometry Maths Page 34 of 58 Website: www.vidhyarjan.com Email: contact@vidhyarjan.com Mobile: 9999 249717 Head Office: 1/3-H-A-2, Street # 6, East Azad Nagar, Delhi-110051 (One Km from ‘Welcome’ Metro Station) Substituting the value of , in equation (1), we obtain Question 13: In the following cases, determine whether the given planes are parallel or perpendicular, and in case they are neither, find the angles between them. (a) (b) (c) (d) (e) Answer The direction ratios of normal to the plane, , are a1, b1, c1 and . The angle between L1 and L2 is given by, (a) The equations of the planes are 7x + 5y + 6z + 30 = 0 and 3x − y − 10z + 4 = 0 Here, a1 = 7, b1 =5, c1 = 6
  • 35. Class XII Chapter 11 – Three Dimensional Geometry Maths Page 35 of 58 Website: www.vidhyarjan.com Email: contact@vidhyarjan.com Mobile: 9999 249717 Head Office: 1/3-H-A-2, Street # 6, East Azad Nagar, Delhi-110051 (One Km from ‘Welcome’ Metro Station) Therefore, the given planes are not perpendicular. It can be seen that, Therefore, the given planes are not parallel. The angle between them is given by, (b) The equations of the planes are and Here, and Thus, the given planes are perpendicular to each other. (c) The equations of the given planes are and Here, and Thus, the given planes are not perpendicular to each other.
  • 36. Class XII Chapter 11 – Three Dimensional Geometry Maths Page 36 of 58 Website: www.vidhyarjan.com Email: contact@vidhyarjan.com Mobile: 9999 249717 Head Office: 1/3-H-A-2, Street # 6, East Azad Nagar, Delhi-110051 (One Km from ‘Welcome’ Metro Station) ∴ Thus, the given planes are parallel to each other. (d) The equations of the planes are and Here, and ∴ Thus, the given lines are parallel to each other. (e) The equations of the given planes are and Here, and Therefore, the given lines are not perpendicular to each other. ∴ Therefore, the given lines are not parallel to each other. The angle between the planes is given by,
  • 37. Class XII Chapter 11 – Three Dimensional Geometry Maths Page 37 of 58 Website: www.vidhyarjan.com Email: contact@vidhyarjan.com Mobile: 9999 249717 Head Office: 1/3-H-A-2, Street # 6, East Azad Nagar, Delhi-110051 (One Km from ‘Welcome’ Metro Station) Question 14: In the following cases, find the distance of each of the given points from the corresponding given plane. Point Plane (a) (0, 0, 0) (b) (3, −2, 1) (c) (2, 3, −5) (d) (−6, 0, 0) Answer It is known that the distance between a point, p(x1, y1, z1), and a plane, Ax + By + Cz = D, is given by, (a) The given point is (0, 0, 0) and the plane is (b) The given point is (3, − 2, 1) and the plane is ∴ (c) The given point is (2, 3, −5) and the plane is (d) The given point is (−6, 0, 0) and the plane is
  • 38. Class XII Chapter 11 – Three Dimensional Geometry Maths Page 38 of 58 Website: www.vidhyarjan.com Email: contact@vidhyarjan.com Mobile: 9999 249717 Head Office: 1/3-H-A-2, Street # 6, East Azad Nagar, Delhi-110051 (One Km from ‘Welcome’ Metro Station)
  • 39. Class XII Chapter 11 – Three Dimensional Geometry Maths Page 39 of 58 Website: www.vidhyarjan.com Email: contact@vidhyarjan.com Mobile: 9999 249717 Head Office: 1/3-H-A-2, Street # 6, East Azad Nagar, Delhi-110051 (One Km from ‘Welcome’ Metro Station) Miscellaneous Solutions Question 1: Show that the line joining the origin to the point (2, 1, 1) is perpendicular to the line determined by the points (3, 5, −1), (4, 3, −1). Answer Let OA be the line joining the origin, O (0, 0, 0), and the point, A (2, 1, 1). Also, let BC be the line joining the points, B (3, 5, −1) and C (4, 3, −1). The direction ratios of OA are 2, 1, and 1 and of BC are (4 − 3) = 1, (3 − 5) = −2, and (−1 + 1) = 0 OA is perpendicular to BC, if a1a2 + b1b2 + c1c2 = 0 ∴ a1a2 + b1b2 + c1c2 = 2 × 1 + 1 (−2) + 1 ×0 = 2 − 2 = 0 Thus, OA is perpendicular to BC. Question 2: If l1, m1, n1 and l2, m2, n2 are the direction cosines of two mutually perpendicular lines, show that the direction cosines of the line perpendicular to both of these are m1n2 − m2n1, n1l2 − n2l1, l1m2 − l2m1. Answer It is given that l1, m1, n1 and l2, m2, n2 are the direction cosines of two mutually perpendicular lines. Therefore, Let l, m, n be the direction cosines of the line which is perpendicular to the line with direction cosines l1, m1, n1 and l2, m2, n2.
  • 40. Class XII Chapter 11 – Three Dimensional Geometry Maths Page 40 of 58 Website: www.vidhyarjan.com Email: contact@vidhyarjan.com Mobile: 9999 249717 Head Office: 1/3-H-A-2, Street # 6, East Azad Nagar, Delhi-110051 (One Km from ‘Welcome’ Metro Station) l, m, n are the direction cosines of the line. ∴l2 + m2 + n2 = 1 … (5) It is known that, ∴ Substituting the values from equations (5) and (6) in equation (4), we obtain Thus, the direction cosines of the required line are
  • 41. Class XII Chapter 11 – Three Dimensional Geometry Maths Page 41 of 58 Website: www.vidhyarjan.com Email: contact@vidhyarjan.com Mobile: 9999 249717 Head Office: 1/3-H-A-2, Street # 6, East Azad Nagar, Delhi-110051 (One Km from ‘Welcome’ Metro Station) Question 3: Find the angle between the lines whose direction ratios are a, b, c and b − c, c − a, a − b. Answer The angle Q between the lines with direction cosines, a, b, c and b − c, c − a, a − b, is given by, Thus, the angle between the lines is 90°. Question 4: Find the equation of a line parallel to x-axis and passing through the origin. Answer The line parallel to x-axis and passing through the origin is x-axis itself. Let A be a point on x-axis. Therefore, the coordinates of A are given by (a, 0, 0), where a ∈ R. Direction ratios of OA are (a − 0) = a, 0, 0 The equation of OA is given by, Thus, the equation of line parallel to x-axis and passing through origin is
  • 42. Class XII Chapter 11 – Three Dimensional Geometry Maths Page 42 of 58 Website: www.vidhyarjan.com Email: contact@vidhyarjan.com Mobile: 9999 249717 Head Office: 1/3-H-A-2, Street # 6, East Azad Nagar, Delhi-110051 (One Km from ‘Welcome’ Metro Station) Question 5: If the coordinates of the points A, B, C, D be (1, 2, 3), (4, 5, 7), (−4, 3, −6) and (2, 9, 2) respectively, then find the angle between the lines AB and CD. Answer The coordinates of A, B, C, and D are (1, 2, 3), (4, 5, 7), (−4, 3, −6), and (2, 9, 2) respectively. The direction ratios of AB are (4 − 1) = 3, (5 − 2) = 3, and (7 − 3) = 4 The direction ratios of CD are (2 −(− 4)) = 6, (9 − 3) = 6, and (2 −(−6)) = 8 It can be seen that, Therefore, AB is parallel to CD. Thus, the angle between AB and CD is either 0° or 180°. sQuestion 6: If the lines and are perpendicular, find the value of k. Answer The direction of ratios of the lines, and , are −3, 2k, 2 and 3k, 1, −5 respectively. It is known that two lines with direction ratios, a1, b1, c1 and a2, b2, c2, are perpendicular, if a1a2 + b1b2 + c1c2 = 0
  • 43. Class XII Chapter 11 – Three Dimensional Geometry Maths Page 43 of 58 Website: www.vidhyarjan.com Email: contact@vidhyarjan.com Mobile: 9999 249717 Head Office: 1/3-H-A-2, Street # 6, East Azad Nagar, Delhi-110051 (One Km from ‘Welcome’ Metro Station) Therefore, for , the given lines are perpendicular to each other. Question 7: Find the vector equation of the plane passing through (1, 2, 3) and perpendicular to the plane Answer The position vector of the point (1, 2, 3) is The direction ratios of the normal to the plane, , are 1, 2, and −5 and the normal vector is The equation of a line passing through a point and perpendicular to the given plane is given by, Question 8: Find the equation of the plane passing through (a, b, c) and parallel to the plane Answer Any plane parallel to the plane, , is of the form The plane passes through the point (a, b, c). Therefore, the position vector of this point is Therefore, equation (1) becomes
  • 44. Class XII Chapter 11 – Three Dimensional Geometry Maths Page 44 of 58 Website: www.vidhyarjan.com Email: contact@vidhyarjan.com Mobile: 9999 249717 Head Office: 1/3-H-A-2, Street # 6, East Azad Nagar, Delhi-110051 (One Km from ‘Welcome’ Metro Station) Substituting in equation (1), we obtain This is the vector equation of the required plane. Substituting in equation (2), we obtain Question 9: Find the shortest distance between lines and . Answer The given lines are It is known that the shortest distance between two lines, and , is given by Comparing to equations (1) and (2), we obtain
  • 45. Class XII Chapter 11 – Three Dimensional Geometry Maths Page 45 of 58 Website: www.vidhyarjan.com Email: contact@vidhyarjan.com Mobile: 9999 249717 Head Office: 1/3-H-A-2, Street # 6, East Azad Nagar, Delhi-110051 (One Km from ‘Welcome’ Metro Station) Substituting all the values in equation (1), we obtain Therefore, the shortest distance between the two given lines is 9 units. Question 10: Find the coordinates of the point where the line through (5, 1, 6) and (3, 4, 1) crosses the YZ-plane Answer It is known that the equation of the line passing through the points, (x1, y1, z1) and (x2, y2, z2), is The line passing through the points, (5, 1, 6) and (3, 4, 1), is given by, Any point on the line is of the form (5 − 2k, 3k + 1, 6 −5k). The equation of YZ-plane is x = 0 Since the line passes through YZ-plane, 5 − 2k = 0
  • 46. Class XII Chapter 11 – Three Dimensional Geometry Maths Page 46 of 58 Website: www.vidhyarjan.com Email: contact@vidhyarjan.com Mobile: 9999 249717 Head Office: 1/3-H-A-2, Street # 6, East Azad Nagar, Delhi-110051 (One Km from ‘Welcome’ Metro Station) Therefore, the required point is . Question 11: Find the coordinates of the point where the line through (5, 1, 6) and (3, 4, 1) crosses the ZX − plane. Answer It is known that the equation of the line passing through the points, (x1, y1, z1) and (x2, y2, z2), is The line passing through the points, (5, 1, 6) and (3, 4, 1), is given by, Any point on the line is of the form (5 − 2k, 3k + 1, 6 −5k). Since the line passes through ZX-plane, Therefore, the required point is .
  • 47. Class XII Chapter 11 – Three Dimensional Geometry Maths Page 47 of 58 Website: www.vidhyarjan.com Email: contact@vidhyarjan.com Mobile: 9999 249717 Head Office: 1/3-H-A-2, Street # 6, East Azad Nagar, Delhi-110051 (One Km from ‘Welcome’ Metro Station) Question 12: Find the coordinates of the point where the line through (3, −4, −5) and (2, − 3, 1) crosses the plane 2x + y + z = 7). Answer It is known that the equation of the line through the points, (x1, y1, z1) and (x2, y2, z2), is Since the line passes through the points, (3, −4, −5) and (2, −3, 1), its equation is given by, Therefore, any point on the line is of the form (3 − k, k − 4, 6k − 5). This point lies on the plane, 2x + y + z = 7 ∴ 2 (3 − k) + (k − 4) + (6k − 5) = 7 Hence, the coordinates of the required point are (3 − 2, 2 − 4, 6 × 2 − 5) i.e., (1, −2, 7). Question 13: Find the equation of the plane passing through the point (−1, 3, 2) and perpendicular to each of the planes x + 2y + 3z = 5 and 3x + 3y + z = 0. Answer The equation of the plane passing through the point (−1, 3, 2) is a (x + 1) + b (y − 3) + c (z − 2) = 0 … (1) where, a, b, c are the direction ratios of normal to the plane.
  • 48. Class XII Chapter 11 – Three Dimensional Geometry Maths Page 48 of 58 Website: www.vidhyarjan.com Email: contact@vidhyarjan.com Mobile: 9999 249717 Head Office: 1/3-H-A-2, Street # 6, East Azad Nagar, Delhi-110051 (One Km from ‘Welcome’ Metro Station) It is known that two planes, and , are perpendicular, if Plane (1) is perpendicular to the plane, x + 2y + 3z = 5 Also, plane (1) is perpendicular to the plane, 3x + 3y + z = 0 From equations (2) and (3), we obtain Substituting the values of a, b, and c in equation (1), we obtain This is the required equation of the plane. Question 14: If the points (1, 1, p) and (−3, 0, 1) be equidistant from the plane , then find the value of p. Answer The position vector through the point (1, 1, p) is Similarly, the position vector through the point (−3, 0, 1) is
  • 49. Class XII Chapter 11 – Three Dimensional Geometry Maths Page 49 of 58 Website: www.vidhyarjan.com Email: contact@vidhyarjan.com Mobile: 9999 249717 Head Office: 1/3-H-A-2, Street # 6, East Azad Nagar, Delhi-110051 (One Km from ‘Welcome’ Metro Station) The equation of the given plane is It is known that the perpendicular distance between a point whose position vector is and the plane, is given by, Here, and d Therefore, the distance between the point (1, 1, p) and the given plane is Similarly, the distance between the point (−3, 0, 1) and the given plane is It is given that the distance between the required plane and the points, (1, 1, p) and (−3, 0, 1), is equal. ∴ D1 = D2
  • 50. Class XII Chapter 11 – Three Dimensional Geometry Maths Page 50 of 58 Website: www.vidhyarjan.com Email: contact@vidhyarjan.com Mobile: 9999 249717 Head Office: 1/3-H-A-2, Street # 6, East Azad Nagar, Delhi-110051 (One Km from ‘Welcome’ Metro Station) Question 15: Find the equation of the plane passing through the line of intersection of the planes and and parallel to x-axis. Answer The given planes are The equation of any plane passing through the line of intersection of these planes is Its direction ratios are (2λ + 1), (3λ + 1), and (1 − λ). The required plane is parallel to x-axis. Therefore, its normal is perpendicular to x-axis. The direction ratios of x-axis are 1, 0, and 0. Substituting in equation (1), we obtain
  • 51. Class XII Chapter 11 – Three Dimensional Geometry Maths Page 51 of 58 Website: www.vidhyarjan.com Email: contact@vidhyarjan.com Mobile: 9999 249717 Head Office: 1/3-H-A-2, Street # 6, East Azad Nagar, Delhi-110051 (One Km from ‘Welcome’ Metro Station) Therefore, its Cartesian equation is y − 3z + 6 = 0 This is the equation of the required plane. Question 16: If O be the origin and the coordinates of P be (1, 2, −3), then find the equation of the plane passing through P and perpendicular to OP. Answer The coordinates of the points, O and P, are (0, 0, 0) and (1, 2, −3) respectively. Therefore, the direction ratios of OP are (1 − 0) = 1, (2 − 0) = 2, and (−3 − 0) = −3 It is known that the equation of the plane passing through the point (x1, y1 z1) is where, a, b, and c are the direction ratios of normal. Here, the direction ratios of normal are 1, 2, and −3 and the point P is (1, 2, −3). Thus, the equation of the required plane is Question 17: Find the equation of the plane which contains the line of intersection of the planes , and which is perpendicular to the plane . Answer The equations of the given planes are The equation of the plane passing through the line intersection of the plane given in equation (1) and equation (2) is
  • 52. Class XII Chapter 11 – Three Dimensional Geometry Maths Page 52 of 58 Website: www.vidhyarjan.com Email: contact@vidhyarjan.com Mobile: 9999 249717 Head Office: 1/3-H-A-2, Street # 6, East Azad Nagar, Delhi-110051 (One Km from ‘Welcome’ Metro Station) The plane in equation (3) is perpendicular to the plane, Substituting in equation (3), we obtain This is the vector equation of the required plane. The Cartesian equation of this plane can be obtained by substituting in equation (3). Question 18: Find the distance of the point (−1, −5, −10) from the point of intersection of the line and the plane . Answer The equation of the given line is The equation of the given plane is Substituting the value of from equation (1) in equation (2), we obtain
  • 53. Class XII Chapter 11 – Three Dimensional Geometry Maths Page 53 of 58 Website: www.vidhyarjan.com Email: contact@vidhyarjan.com Mobile: 9999 249717 Head Office: 1/3-H-A-2, Street # 6, East Azad Nagar, Delhi-110051 (One Km from ‘Welcome’ Metro Station) Substituting this value in equation (1), we obtain the equation of the line as This means that the position vector of the point of intersection of the line and the plane is This shows that the point of intersection of the given line and plane is given by the coordinates, (2, −1, 2). The point is (−1, −5, −10). The distance d between the points, (2, −1, 2) and (−1, −5, −10), is Question 19: Find the vector equation of the line passing through (1, 2, 3) and parallel to the planes and . Answer Let the required line be parallel to vector given by, The position vector of the point (1, 2, 3) is The equation of line passing through (1, 2, 3) and parallel to is given by, The equations of the given planes are
  • 54. Class XII Chapter 11 – Three Dimensional Geometry Maths Page 54 of 58 Website: www.vidhyarjan.com Email: contact@vidhyarjan.com Mobile: 9999 249717 Head Office: 1/3-H-A-2, Street # 6, East Azad Nagar, Delhi-110051 (One Km from ‘Welcome’ Metro Station) The line in equation (1) and plane in equation (2) are parallel. Therefore, the normal to the plane of equation (2) and the given line are perpendicular. From equations (4) and (5), we obtain Therefore, the direction ratios of are −3, 5, and 4. Substituting the value of in equation (1), we obtain This is the equation of the required line. Question 20: Find the vector equation of the line passing through the point (1, 2, − 4) and perpendicular to the two lines: Answer Let the required line be parallel to the vector given by,
  • 55. Class XII Chapter 11 – Three Dimensional Geometry Maths Page 55 of 58 Website: www.vidhyarjan.com Email: contact@vidhyarjan.com Mobile: 9999 249717 Head Office: 1/3-H-A-2, Street # 6, East Azad Nagar, Delhi-110051 (One Km from ‘Welcome’ Metro Station) The position vector of the point (1, 2, − 4) is The equation of the line passing through (1, 2, −4) and parallel to vector is The equations of the lines are Line (1) and line (2) are perpendicular to each other. Also, line (1) and line (3) are perpendicular to each other. From equations (4) and (5), we obtain ∴Direction ratios of are 2, 3, and 6. Substituting in equation (1), we obtain This is the equation of the required line.
  • 56. Class XII Chapter 11 – Three Dimensional Geometry Maths Page 56 of 58 Website: www.vidhyarjan.com Email: contact@vidhyarjan.com Mobile: 9999 249717 Head Office: 1/3-H-A-2, Street # 6, East Azad Nagar, Delhi-110051 (One Km from ‘Welcome’ Metro Station) Question 21: Prove that if a plane has the intercepts a, b, c and is at a distance of P units from the origin, then Answer The equation of a plane having intercepts a, b, c with x, y, and z axes respectively is given by, The distance (p) of the plane from the origin is given by, Question 22: Distance between the two planes: and is (A)2 units (B)4 units (C)8 units (D) Answer The equations of the planes are
  • 57. Class XII Chapter 11 – Three Dimensional Geometry Maths Page 57 of 58 Website: www.vidhyarjan.com Email: contact@vidhyarjan.com Mobile: 9999 249717 Head Office: 1/3-H-A-2, Street # 6, East Azad Nagar, Delhi-110051 (One Km from ‘Welcome’ Metro Station) It can be seen that the given planes are parallel. It is known that the distance between two parallel planes, ax + by + cz = d1 and ax + by + cz = d2, is given by, Thus, the distance between the lines is units. Hence, the correct answer is D. Question 23: The planes: 2x − y + 4z = 5 and 5x − 2.5y + 10z = 6 are (A) Perpendicular (B) Parallel (C) intersect y-axis (C) passes through Answer The equations of the planes are 2x − y + 4z = 5 … (1) 5x − 2.5y + 10z = 6 … (2) It can be seen that,
  • 58. Class XII Chapter 11 – Three Dimensional Geometry Maths Page 58 of 58 Website: www.vidhyarjan.com Email: contact@vidhyarjan.com Mobile: 9999 249717 Head Office: 1/3-H-A-2, Street # 6, East Azad Nagar, Delhi-110051 (One Km from ‘Welcome’ Metro Station) ∴ Therefore, the given planes are parallel. Hence, the correct answer is B.